Sunteți pe pagina 1din 128

M.Sc.

(MATHEMATICS)
MAL-513

MECHANICS

DIRECTORATE OF DISTANCE EDUCATION


GURU JAMBHESHWAR UNIVERSITY OF SCIENCE & TECHNOLOGY

HISAR-125001

TABLE OF CONTENTS
LESSON No. 1 2 3 4 5 6 NAME OF THE LESSON Moment of Inertia-1 Moment of Inertia-2 Generalized co-ordinates and Lagranges Equations Hamiltons Equations of Motion Canonical Transformations Attraction and Potential

Author
Prof. Kuldip Singh Department of Mathematics GJUS & T Hisar

Lesson: 1 Moment of Inertia-1


1.1 Some definations:Inertia:- Inertia of a body is the inability of the body to change by itself its state of rest or state of uniform motion along a straight line. Inertia of motion:- It is the inability of a body to change by itself its state of motion. Moment of Inertia:- A quantity that measures the inertia of rotational motion of body is called rotational inertia or moment of inertia of body. M.I. is rotational analogue of mass in linear motion. We shall denote it by I. Let there are n particles of masses mi, then moment of inertia of the system is O
2 I = m1 d1 + m 2 d 2 + ... + m n d 2 2 n

d1 d2 d3 dn mn m2

m1 m3

m i d i2
i =1

I = md2

L where di are the distances of particles from the axis. (i) M.I. in three dimension: - Let us consider a three dimensional body of volume
V. Let OL be axis of rotation. Consider an infinitesimal small element of mass dm, then mass of small element dm = dv where dv = volume of infinitesimal small element and

O d dm

is the density of material. Then moment of inertia of


body is I=

d
v

d2

L
2

or

I=

d
v

dv

(ii) M.I. in two dimension

O d dm

Here mass of small element dm = dS and moment of inertia is or I = d m d 2


S

I=

d
S

dS O

where dS = surface area of small element


(iii) M.I. in one dimension

Consider a body (a line or curve) in one dimension. Consider a small element of length ds and mass dm. Then mass of small element, dm = ds M.I. of small element = dmd2
M.I. of body

d L

ds

I = d md 2
s

or

I = d 2 ds
s

Radius of gyration:- Radius of gyration of a body about a given axis is the

distance of point P from the axis where its whole mass of body were concentrated, the body shall have the same moment of inertia as it has with the actual distribution of mass. This distance is represented by K. When K is radius of gyration, I = I

2 2 MK2 = m (r12 + r2 + ... + rn )

2 2 mn (r12 + r2 + ... + rn ) n

MK2 =

M (r12 + r22 + .... + rn2 ) n

K=

2 2 r12 + r2 + ... + rn n

where n is the number of particles of the body, each of mass m and r1, r2 rn be the perpendicular distances of these particles from axis of rotation. Where M = m n = total mass of body. m r1 m r2 r3 K m M P

Hence radius of gyration of a body about a given axis is equal to root mean square distance of the constituent particles of the body from the given axis.
Example:-M.I. of a uniform rod of length 2a about an axis passing through one end and to the rod:-

O 2a

A L Let M = mass of rod of length 2a.

OL = axis of rotation passing through one end A and to rod. Mass per unit length of rod = M 2a

Consider a small element of breadth x at a distance x from end A. Mass of this small element = M x 2a
M x x2 2a

M.I. of small element about axis OL or AL =


2a

M.I. of rod about OL =

2a x
0

dx

M x3 I= 2a 3 0

2a

I=

M 8a 4 = Ma2 2a 3 3 4 Ma2 3

IOL =

Example:- M.I. of a rod about an axis passing through mid-point and to rod

Here LL is the axis of rotation passing through mid-point 0 of rod having length 2a. Consider a small element of breadth x at a distance x from mid-point of rod O. M x 2a M x. x2 2a A 0

L 2a x

x L

Mass of this small element =

M.I. of small element about LL = M.I. of rod about LL =


M 2 x dx 2a a
a a

a 2M 2 M x3 ILL = x dx = 2a a 3 0 0

M 3 Ma 2 a = 3a 3 1 Ma 2 3

ILL =

Example:-M.I. of a rectangular lamina about an axis (line) passing through centre and parallel to one side

D N A G x

PQ

C L

Let ABCD be a rectangular lamina of mass M and NL be the line about which M.I. is to be calculated. Let AB = 2a, BC = 2b Mass per unit area of lamina = M 4ab

Consider an elementary strip PQ of length (BC = 2b) and breadth x and at a distance x from G and parallel to AD. Mass of elementary strip = = M.I. of this strip about NL = M . 2b x 4ab M x 2a b2 (Mass of strip) 3

M b2 = x. 2a 3 M.I. of rectangular lamina about NL = M b2 Mb 2 2a = 2a 3 3


M.I. of rectangular

M b2 x 2a 3 a

I=

Example:-

lamina about a line to lamina and passing through centre :

L D G A d x B y C

Let GL = axis of rotation passing through centre G and to lamina ABCD. Consider a small element of surface area S = xy Here distance of small element from axis GL, d =

x 2 + y2

Mass of small element = x y

M.I. of this small element about GL = x y (x2 + y2) M.I. of lamina =

ba

(x
ba

b a

+ y2)dx dy

= 4 ( x 2 + y2) dx dy
00
b b x3 a3 2 = 4 + xy dy = 4 + ay 2 dy 3 3 0 0 o a

a3 ay 3 = 4 y + 3 3 0 = I=

4a 2 4ab 2 (a b + b 3 ) = (a + b2) 3 3 [using mass of lamina M = 4 ab]

M 2 (a + b2) 3

1.2 Moments and products of inertia about co-ordinate axes:(I) For a particle system

Q z m P(x,y,z) O(0,0,0 ) d x
Consider a single particle P of mass m having co-ordinates (x, y, z) Here d = distance of particle P of mass m from z-axis = PQ = OP = x 2 + y2

y P(x,y,0)

Therefore, M.I. of particle of mass m about z-axis = md2 = m (x2 + y2)

M.I. of system of particles about z-axis Ioz = md2 = m(x2 + y2) And Standard notation for M.I about z-axis is C, i.e., C = m (x2 + y2) = Ioz Similarly, we can obtain M. I. about x and y-axis which are denoted as under: About x-axis, A = (y2 + z2) = Iox About y-axis, B = m (z2 + x2) = Ioy

Product of Inertia
The quantities D = myz E = mzx and F = mxy are called products of inertia w.r.t. pair of axes (oy, oz), (oz, ox) and (ox, oy) respectively.

For a continuous body:


The M.I. about z-axis, x-axis and y-axis are defined as under C=

(x
V V

+ y 2 ) dx dy dz

A = (y2 + z2)dx dy dz B = (z2 + x2)dx dy dz


V

Similarly, the products of inertia w.r.t. pair of axes (oy, oz), (oz, ox) and (ox, oy) respectively are as under D = = yz dv ,E = zx dV and F = = xy dV
V V V

For laminas in xy plane, we put z = 0, then A= B= C=

y
S

dxdy dx dy

x
S S

(x

+ y2)dxdy

D = E = 0, F =

xy dx dy
S

1.3 M.I. of a body about a line (an axis) whose direction cosines are <, , > :-

Let a is a unit vector in axis OL whose direction cosines are <, , >. Then a = + + k i j ...(1)

<, , > N d a r P(x, y, z)


y

Let P(x, y, z) be any point(particle) of mass of the body. 0 Then its position vector r is given by OP = r = x + y + zk i j
distance of P from OL, d = PN = OP sin = | r a |
x

(2)

(3)

d = ( x + y + zk ) ( + + k ) i j i j i j = |(y z) + (z x ) + (x y)k | = ( y z) 2 + (z x ) 2 + (x y) 2 2 ( y 2 + z 2 ) + 2 (z 2 + x 2 ) + 2 ( x 2 + y 2 ) 2yz 2xy 2xy

d=

Therefore, M.I. of body about an axis whose direction cosine are , , is

I OL = m{2 ( y 2 + z 2 ) + 2 (z 2 + x 2 ) + 2 ( x 2 + y 2 )
2yz 2xz 2xy}

IOL = A2 + B2 + C2 2D 2E 2F

1.4 Kinetic Energy (K.E.) of a body rotating about O:-

Let axis of rotation be OL through O, then angular velocity about OL is w = wa


Then K.E., T =

1 m ( v. v) 2

= T=

1 m | v |2 2
[ v = w r = wa r

1 = m | a r |2 w 2 2

10

1 2 w md2 2 T= 1 2 w IOL 2

[using equation (3)]

This is the required expression for kinetic energy in terms of moment of inertia

1.5 Parallel axis theorem Statement:- For a body of mass M, we have


C = C + Md2 where C = M. I of body about a line GL through C.G.( centre of mass) and parallel to z-axis C = M.I. of body about z-axis (i.e. a line parallel to GL) and at a distance d from GL. z d z (x,y,z) P(x,y,z) G ( x , y, z ) y x o y L

Proof :
N

(0,0,0)

x Let M = Mass of body and P is any point whose co-ordinates w.r.t. oxyz are (x,y,z), G is the centre of mass whose co-ordinates w.r.t. oxyz are ( ( x , y, z ) ). Let us introduce a new co-ordinate system Gxyz through G and Co-ordinates of P w.r.t. this system are (x,y,z). Let rG be the position vector of G and ri position

vector of mass mi w.r.t. oxyz system. Now by definition of centre of mass of body,
m i ri rG = M

11

when centre of mass cocides with origin at G w.r.t new co-ordinate system G xyz, we have rG = 0 . Therefor m i ri =0 M
mi ri = 0

m x ' = 0, M

my' = 0, M

mz' =0 M

where

and So, we have

r ' = x ' + y' + z ' k i j r = x i + yj + zk

m x = my = mz = 0

(1)

Now

d2 = (GN)2 = (OG)2 (ON)2 = x 2 + y 2 + z 2 z 2 = x 2 + y2 (2)

Co-ordinates of P w.r.t. (ox, oy, oz) axes is (x, y, z) Co-ordinates of P w.r.t. (Gx, Gy, Gz) axes is (x, y, z) Then x = x + x ' , y = y + y' , z = z + z Thus, M.I. about z-axis is C = m(x2 + y2) = m[( x + x ' ) 2 + ( y + y' ) 2 ]

C = m [ x 2 + x '2 +2x x '+ y 2 + y' 2 +2 y y' ] = m(x2 + y2) + m( x 2 + y 2 ) + 2x mx + 2 y m y C = m (x2 + y2) + ( x 2 + y 2 ) m + 0 [from (1)]

C = C + Md2 A = A + Md2 B = B + Md2

[using (2) and m = M, total mass]

Similarly, M.I. about x and y-axis are given by

where d is perpendicular distance of P from x and y-axis

12

For Product of Inertia


Here Product of Inertia w.r.t. pair (ox,oy) is F = mxy = m( x + x ' )( y + y' ) = m( x y + x ' y + x y'+ x ' y' ) = m x y + x y m + x m y'+ y m x = F + M x y + 0

[using (1)]

F = F + M x y

Similarly, for products of Inertia w.r.t. pair (oy, oz) and (oz, ox) respectively, we have D = D + M y z and E = E + M z x .

1.6 Perpendicular axis theorem


(For Two dimensional bodies or mass distribution)
Statement:- The M.I. of a plane mass distribution (lamina) w.r.t. any normal axis is

equal to sum of the moments of inertia about any two axis in the plane of mass distribution (lamina) and passing through the intersection of the normal with the lamina.
Proof :

z Let ox, oy are the axes in the

plane of lamina and oz be the normal axis, i.e., xy is the plane of lamina. Let C is the M.I. about axis, i.e., oz axis Here to prove C = A + B By definition, M.I. of plane lamina about z-axis, C=
=

(x
S

+ y2) dS [for a continuous body]

x
S

dS +

y
S

dS

C=B+A C = m (x2 + y2) = m x2 + my2

For mass distribution,

13

C=B+A

For two dimensional body, D = E = 0 and F = mxy.


Converse of perpendicular axis theorem :

Given

C=A+B

To prove it is a plane lamina.


Proof:- Here A = m(y2 + z2)

B = m(z2 + x2), C = M(x2 + y2) Now given C = A + B


(x2 + y2) = m (y2 + z2) + m(z2 + x2)

= m(y2 + 2z2 + x2)


mx2 + my2 = my2 + 2mz2 + mx2

2mz2 = 0
mz2 = 0 for all distribution of mass.

For a single particle of mass m, mz2 = 0 z = 0 as m 0


It is a plane mass distribution or it is a plane lamina.

1.7 Angular momentum of a rigid body about a fixed point and about a fixed axis:- The turning effect of a particle about the axis of rotation is called
angular Momentum. Let O be the fixed point and OL be an axis passing through the fixed point. w = angular velocity about OL r = position vector of P(x, y, z)
r = OP = x + y + z k i j Also linear velocity of P, v = w r

P(m) r

O (fixed point) (1)

The angular momentum of body about O is H = ( r mv) = [ r m( w r )]

(2)

14

H = m [ r ( w r )] = m [( r .r ) w ( r .w ) r ]

[ AX(B C) = (A. C) B (A . B)C]


= m r 2 w ( r .w ) r ] H = (m r 2 ) w m( r .w ) r H = h1 + h 2 + h 3 k i j

(3)

If

w = w1 + w 2 + w 3 k i j Then r . w = w 1x + w 2 y + w 3 z from (3),

(4)

i j i j h1 + h 2 + h 3 k = (m r 2 ) ( w 1 + w 2 + w 3 k ) m(w1x + w2y + w3z) ( x + y + z k ) i j Equating coefficients of on both sides, i h1 = m (x2 + y2 + z2) w1 m(w1x + w2y + w3z)x = m(y2 + z2)w1 + m x2w1 mw1 x2 m(w2y + w3z)x = m(y2 + z2) w1 (m xy) w2 (m xz)w3

h1 = Aw1 Fw2 Ew3 h2 = Bw2 Dw3 Fw1 h3 = Cw3 Ew1 Dw2


h1 A F E w 1 h = F B D w 2 2 h 3 E D C w 3

Similarly,

(5)

Inertia matrix (symmetric 3 3 matrix)

1.8 Principal axis and their determination


the axis is known as principal axis.

Definition :- If the axis of rotation w is parallel to the angular momentum H , then

15 w =| w | a = w a H =| H | a H = nw

If

H = n w , where n is a constant

1.8.1 Theorem:- Prove that in general, there are three principal axes through a
point of rigid body.

Proof : For principal axis, H = n w H = nw H = Ha , w = wa Let


where a is a unit vector along principal axis of body through O. By definition of H , H = ( r mv) H = (mr 2 ) w m( r . w ) r Using H = n w , we get n w = (mr 2 ) w m( r .w ) r Using w = w a , n w a = mr 2 wa m( r .w a ) r Cancelling w on both sides & rearranging, (mr 2 n )a = m( r .a ) r Let r = x + y + z k , i j a = + + k i j

(1)

(2) (3)

where <, , > are direction cosine of principal axis. (mr2 n) ( + + k ) = m[x + y + z) ( x + y + z k ) ] i j i j

Equating coefficients of on both sides, i Similarly [m(x2 + y2 + z2)n] = m(x2 + xy + xz) [m(y2 + z2)n] = m[xy + xz] (A n) F E = 0 (B n) D F = 0 (4) [canceling m x2 on both sides]

16

(C n) E D = 0 or (A n) F E = 0 F + (Bn) D = 0 E D + (C n) = 0 Equation (5) has a non-zero solution only if An F E F Bn D E D Cn =0 (6) (5)

This determinental equation is a cubic in n and it is called characteristic equation of symmetric inertia matrix. This characterstic equation has three roots n1, n2, n3 (say), so n1, n2, n3 are real. Corresponding to n = (n1, n2, n3) (solving equation (5) or (6) for <, , >). Let the values of (, , ) be (1, 1, 1) n = n1 (2, 2, 2) n = n2 (3, 3, 3) n = n3 These three sets of value determine three principal axes a 1 , a 2 , a 3 given by a p = p + p + p k where p = 1, 2, 3. i j

1.8.2 Theorem :- Three principal axes through a point of a rigid body are mutually
orthogonal.

Proof : Let the three principal axes corresponding to roots n1, n2, n3 of
characteristic equation An F F Bn

E
be a 1 , a 2 , a 3 .

E D = 0 Cn

Let n 1 , n 2 , n 3 are all different.

17

Then from equation,

(m r2n) a = m( r . a ) r

We have

(mr2 n1) a 1 = m( r . a 1 ) r (mr2 n2) a 2 = m( r .a 2 ) r (mr2 n3) a 3 = m( r . a 3 ) r

(1) (2) (3)

Multiply scalarly equation (1) by a 2 and equation (2) with a 1 and then substracts, we get (n1 n2) a 1.a 2 = 0

a 1.a 2 = 0 as n1 n2

Similarly a 2 , a 3 = 0 and a 3 .a 1 = 0

a 1 , a 2 , a 3 are mutually orthogonal.

Remarks :- (i) If n1 n2 n3 then there are exactly three mutually axis through 0.
(ii) If n2 = n3 (i.e. two characteristic roots are equal). There is one principal axis corresponding to n1 through 0. Then every line through 0 & to this a 1 is a principal axis. Infinite set of principal axis with the condition that a 1 is fixed. (iii) If n1 = n2 = n3, then Any three mutually axes through O(centre of sphere) are principal axes. Z a3 a1 a2 0 a1

18

1.9 Moments and products of Inertia about principal axes and hence to find angular momentum of body.
Let a 1 , a 2 , a 3 are the principal axes. Let us take co-ordinates axes along the principal axes. r = OP = Xa 1 + Ya 2 + Za 3 X a1 O a3 P(X,Y,Z) r a2 Y

r2 = X 2 + Y 2 + Z2

From equation

(mr2n) a = m( r . a ) r

We have

(mr2 n1) a 1 = m( r . a 2 ) r (mr2 n2) a 2 = m( r . a 2 ) r (mr2 n3) a 3 = m( r . a 3 ) r


(mr2 n1) a 1 = m[(Xa 1 + Ya 2 + Za 3 ).a 1[Xa 1 + Ya 2 + Za 3 ] = m( Xa 1 + Ya 2 + Za 3 )

(1) (2) (3)

From (1),

Equating coefficients of a 1 a 2 , a 3 ,

m(X2 + Y2 + Z2) n1 = mX2


0 = mXY 0 = mXZ or & n1 = m(Y2 + Z2) = A* F* = 0, E* = 0 (4)

Similarly from (2) & (3), we get n2 = B*, D* = 0, F* = 0 & n3 = C*, E* = 0, D* = 0

19

where A*, B*, C* are M.I. and D*, E*, F* are product of Inertia about principal axes. Inertia matrix for principal axes through O is

n1 0 0

0 n2 0

0 A* 0 0 0 = 0 B* 0 n3 0 0 C *

Definition: Three mutually lines through any point of a body which are such that
the product of inertia about them vanishes are known as principal axes. Expression for angular momentum ( H ) Here D* = E* = F* = 0, then from equation, h1 = Aw1 Fw2 Ew3 we have h1 = A*w1 F*w2 E* w3

h1 = A*w1

[ F* = E* = 0]

Similarly h2 = B* w2, h3 = C* w3 H = h1 a 1 + h 2 a 2 + h 3 a 3 = A* w1 a 1 + B * w 2 a 2 + C * w 3a 3 where (w1, w2, w3) are components of angular velocity about ( a 1 , a 2 , a 3 ). A*, B*, C* are also called principal moments of inertia.

1.10 Momental Ellipsoid:- We


know that M.I., IOL of a body about the line whose d.c.s are <, , > is IOL = I = A2 + B2 + C2 2D

2E 2F

(1)

20

Let P(x,y, z) be any point on OL and OP = R, then R = R ( + + k ) = x + y + zk i j i j =


x , R = y , R = z R (2)

Now let P moves in such a way that IR2 remains constant, then from (1), (2), we get Ax2 + By2 + Cz2 2Dyz 2Ezx 2Fxy = IR2 = constant Since coefficients of x2, y2, z2 i.e. A, B, C all are positive, this equation represents an ellipsoid known as momental ellipsoid.

Example:- A uniform solid rectangular block is of mass m and dimension 2a 2b

2c. Find the equation of the momental ellipsoid for a corner O of the block,
referred to the edges through O as co-ordinates axes and hence determine M.I. about OO where O is the point diagonally opposite to O.

Solution :

z (0, 0, 2c) 0(2a,2b,2c) 2b 2a 0(0,0,0) x (2a,0,0) 2c (0,2b,0) y

Taking x, y, z axes along the edges of lengths 2a, 2b, 2c, we obtain A=

( y
V 2 a 2 b2 c

+ z 2 )dv

(y
0 0 0

+ z2)dz dy dx
2c

z3 = y 2 z + dy dx 3 0 0 0
1 = y 2 2c + 8c 3 dy dx 3 0 0
2a 2 b

2a 2 b

21

4 = . 2c y 2 + c 2 dy dx 3 0 0

2a 2 b

y3 4 = . 2c + c 2 y dx 3 3 0 0
2c 2 3 = (8b + 4c 2b) dx 3 0
2c = 8b (b 2 + c 2 )dx 3 0 = A= 16bc 2 4 (b + c2)2a = (8abc ) (b2 + c2) 3 3
2a

2a

2b

2a

4M 2 (b + c2) 3 4M 2 4M 2 (c + a 2 ), C = (a + b 2 ) 3 3
2 a 2 b2 c 0 0 0

Similarly B = D=

yz dV = yz dz dy dx
v 2a 2 b 2c

z2 = y dy dx 2 0 0 0 = 2
2a2 b 0 0

y(4c
2a 2 b

) dy dx
2a 2b

y2 = 2c y dy dx = 2c dx 2 0 0 0 0
2 2

= c 4b dx = 4b c dx
2 2 2 2 0 0

2a

2a

D = 4b2c2. 2a = (8abc )bc = M bc F = Mab

Similarly E = Mca,

Using these in standard equation of momental ellipsoid, we get

22

4M 2 [(b + c2)x2 + (c2 + a2) y2 + (a2 + b2)z2] 3 2M[bcyz + cazx + ab xy] = IR2 which is required equation of momental ellipsoid. To find M.I. about OO :using x = 2a, y = 2b, z = 2c as O(2a, 2b, 2c) and R2 = 4(a2 + b2 + c2) from (1), (1)

4M 2 [(b + c 2 )4a 2 + (c 2 + a 2 )4b 2 + (a 2 + b 2 )4c 2 ] 8M (b 2 c 2 + c 2 a 2 + a 2 b 2 ) IOO = 3 4( a 2 + b 2 + c 2 ) IOO = IOO = 8M 2(2a 2 b 2 + 2b 2 c 2 + 2c 2 a 2 ) 3(a 2 c 2 + a 2 b 2 + b 2 c 2 ) 3 4(a 2 + b 2 + c 2 ) 2M ( b 2 c 2 + c 2 a 2 + a 2 b 2 ) 3 (a 2 + b 2 + c 2 )

23

Lesson-2

Moment of Inertia-2

2.1 Equimomental Systems:- Two systems are said to be equimomental if they

have equal M.I. about every line in space.


2.1.1 Theorem:- The necessary and sufficient condition for two systems to be

equimomental are : (i) (ii) (iii) They have same total mass. They have same centroid. They have same principal axes.

Proof:- Part A : The condition (i) to (iii) are sufficient. Here we assume that if (i)

to (iii) hold, we shall prove that two systems are equimomental. Let M be the total mass of each system. I system z G h
(<, , >)

II system
(<, , >)

x M

y M

Let G be the common centroid of both the system. Let A*, B*, C* be the principal M.I. about principal axes through G for both the systems. Let be any line in space with d.c. <, , >. We draw a line similar to passing through G. Let h = distance of G from. M.I. about for both the system is I = A*2 + B*2 + C*2 [ Product of inertia about principal axes i.e. D* = E* = F* = 0]

24

So by parallel axes theorem, the M.I. of both the system about is I = I + Mh2

I = A*2 + B*2 + C*2 + Mh2

Hence both the system have same M.I. about any line of space. So they are equimomental.
Part B:- The conditions are necessary. Here we assume that the two systems are

equimomental and derive condition (i) to (iii). Let M1 and M2 be the total masses of the two systems respectively and G1 & G2 are their centroid respectively.
Condition (i)

M1

h M2 G1 G2

Since the systems are equimomental i.e. they have same M.I., I (say) about line G1G2(in particular). Let be the line in space which is parallel to G1 G2 at a distance h. Then by parallel axes theorem, M.I. of Ist system about = I + M1h2 and M.I. of IInd system about = I + M2h2. Since the two systems are equimomental, therefore we have, I + M1 h 2 = I + M 2 h 2

M1 = M2 = M (say)

This implies that both the systems have same total mass.
Condition (ii)

H1

H2

G1 M

G2

25

Let G1H1 and G2H2 be two parallel lines each being to G1 G2. Let I* be the M.I. of either system about a line G1H1 and to G1G2 (through G1) Using parallel axes theorem, M.I. of Ist system about G2H2 = I* + M (G1G2)2 M.I. of IInd system about G2H2 = I* M (G1G2)2 As the systems are equimomental, therefore I* + M (G1G2)2 = I* M(G1G2)2

(G1G2)2 = 0 as M 0 G1 = G2 = G (say) Both the systems have same centroid.

Condition (iii):- Since the two systems are equimomental, they have the same M.I.

about every line through their common centroid. Hence they have same principal axes and principal moments of inertia.
2.2 Coplanar distribution:2.2.1Theorem:- (i) Show that for a two dimensional mass distribution (lamina), one

of the principal axes at O is inclined at an angle to the x-axis through O such that tan 2 = 2F BA

where A, B, F have their usual meanings. (ii) Show that maximum and minimum values of M.I. at O are attained along principal axes. OR
Theorem: - For a 2-D mass distribution (lamina), the value of maximum and

minimum M.I. about lines passing through a point O are attained through principal axes at O.
Proof :-

y x
(x,y) P(x,y)

L N Let us consider an arbitrary particle of mass m at P whose coordinates w. r.t. axes x 0(0, 0)

26

through O are (x,y), then for mass distribution, we have M.I. about x-axis i.e. A = my2 M.I. about y-axis i.e. B = mx2 and Product of inertia F = mxy (1)

We take another set of axes ox, oy such that ox is inclined at an angle with xaxis. Then equation of line ox is given by y = x tan

y cos x sin = 0 , equation of oy is 2

(2)

Changing to +

y sin x cos = 0

y sin + x cos = 0

(3)

Let P(x, y) be co-ordinates of P relative to new system of axes ox, oy, then PL = y = length of from P on ox =
y cos x sin cos 2 + sin 2 (4)

= y cos x sin Similarly x = PN = length of from P on oy = y sin + x cos cos 2 + sin 2

= y sin + x cos Therefore, M.I. of mass distribution (lamina) about ox is Iox = my2 = m(y cos x sin )2 = m(y2 cos2 + x2 sin2 2xy sin cos ) = cos2 my2 + sin2 mx2 2 sin cos mxy = A cos2 + B sin2 F sin2 Similarly M.I. of mass distribution (lamina) about oyis given by

(5)

(6)

27

Ioy = A cos2 + + B sin2 + F sin 2 + 2 2 2 = A sin2 + B cos2 + F sin2 Product of inertia w.r.t pair of axes (ox, oy), Ixy = mxy = m(y sin + x cos) (y cos x sin) Ixy = sin cos my2 sin cos mx2 sin2 mxy + cos2 mxy = A sin cos B sin cos + (cos2 sin2)F = (AB) sin 2 + F cos2 2 (8) (7)

The axes ox, oy will be principal axes if I x y = 0 Using equation (8), 1 (AB) sin2 + F cos2 = 0 2 tan2 = = 2F BA (9)

1 2F tan 1 2 BA

This determines the direction of principal axes relative to co-ordinates axes. We shall now show that maximum/minimum (extreme) values of Iox, Ioy are obtained when is determined from (9), We rewrite, Iox and Ioy as Iox = Ioy = 1 1 (A + B) [(BA) cos 2 + 2 F sin 2] 2 2 1 1 (A +B) + [(BA) cos 2 + 2F sin 2] 2 2

For maximum and minimum value of Iox, Ioy,

28

d (I ox ' ) = 0 d i.e.

and

d (I oy ' ) = 0 d

d [(BA) cos 2 + 2F sin 2] = 0 d (B A)2 sin 2 + 4F cos 2 = 0 tan 2 = 2F BA (11)

Similarly

d [(BA) cos 2 + 2F sin 2] = 0 d (BA) 2 sin 2 + 4F cos 2 = 0

tan 2 =

2F BA

So extreme values of Iox and Ioy are attained for given by equation (11) already obtained in (9). Therefore, the greatest and least values of M.I. for mass distribution (lamina) through O are obtained along the principal axes. The extreme values are obtained as under We have, tan 2 = sin 2 2F = cos 2 B A

sin 2 cos 2 1 = = 2F BA 4F 2 + ( B A ) 2 sin 2 = 2F 4F 2 + ( B A ) 2 BA 4F 2 + ( B A ) 2

& cos 2 = Now writing Iox =

1 1 (A + B) [(BA) cos 2 + 2 F sin 2] 2 2

Using values of cos 2 and sin 2 , we obtain the extreme values of Iox and Ioy as under

29

Iox =

4F 2 1 1 (B A)(B A) (A + B) + 2 2 4F 2 + (B A ) 2 4F 2 + ( B A ) 2 = 1 1 ( B A ) 2 + 4F 2 (A + B) 2 2 4F 2 + ( B A ) 2 1 1 (A + B) [ 4F 2 + (B A) 2 ] 2 2 1 1 (A + B) + [ 4F 2 + (B A) 2 ] 2 2

Similarly Ioy =

Example 1:- A square of side a has particles of masses m, 2m, 3m, 4m at its

vertices. Show that the principal M. I. at centre of the square are 2ma2, 3ma2, 5ma2. Also find the directions of principal axes.
Solution :
y y a a R , 2 2 3m x 0 m a a P , 2 2 x 2m a a Q , 2 2

a a S , 2 2 4m

Taking origin O at the centre of square and axes as shown in the figure, we have A = M.I. of system of particles about x-axis. = A= B= a a a a m i y i2 = m 2 + 2m 2 + 3m 2 + 4m 2 i =1 5 2 ma 2 mixi2 a a a a = m + 3m + 4m + 2 m 2 2 2 2
2 2 2 2 4 2 2 2 2

(1)

30

B=

5 2 ma 2

(2)

C = B + A = 5ma2 a a a a a a a a F = mixiyi = m + 3m + 4m + 2 m 2 2 2 2 2 2 2 2

For a two-dimensional mass distribution, D = E = 0 and

ma 2 2ma 2 3ma 2 4ma 2 3 2 2 F= + 4 + 4 4 = ma 2 ma 4 F= 1 2 ma 2 Iox = A cos2 2F sin cos + B sin2 Ioy = A sin2 + 2F sin cos + B cos2 1 (A B) sin2 + F cos2 2

Let ox, oy be the principal axes at O s. t. xox = . Then, we have

and

Ixy =

Since ox & oy are principal axes, therefore Ixy = 0 1 (AB) sin 2 + F cos 2 = 0 2 tan 2 = 2F BA [ A = B = = 4 5 2 ma ] 2 (3)

(3) cos 2 = 0 2 = 2

Diagonals OR & OS are principal axes.


2 5 1 ma 1 5 2 1 ma 2 2 2 2 + 2 ma 2 2 2

Therefore, IOR =

[using equation (I)]

IOR = 3ma2 5 1 2 1 2 1 5 2 1 I oy ' = 2 ma 2 2. 2 ma 2 + 2 ma 2

and Ios = 2ma2

31

M.I. about z-axis is C = B + A C = IOR+ IOS = 3ma2 + 2ma2 C = 5ma2.

Example 2 :- Show that a uniform rod of mass M is equimomental to three


particles situated one at each end of the rod and one at its middle point, the masses of the particle being M M 2M , and respectively. 6 6 3

Solution:- Let AB = 2a is the length of rod having mass M. L


A M 6 G 2M 3 B M 6

Let m, M2m, m are the masses at A, G, B respectively. This system of particles has same centroid and same total mass M. This system of particles has the same M.I. (i.e. each zero) about AB, passing through common centroid G. Therefore, systems are equimomental.

To find m:- we take M.I. of two systems (one system is rod of mass M) and other
system consists of particles. M.I. of rod about GL = Ma 2 3 M.I. of particles about GL = ma2 + 0 + ma2 = 2ma2 As systems are equimomental, Ma 2 2ma2 = 3 M m= 6 M 2M & M 2m = M = 3 3 So masses of particles at A, G, B are M 2M M , , respectively. 6 3 6

32

Example 3 :- Find equimomental system for a uniform triangular lamina. A Solution:- Let M = Mass of lamina.
Let distance of A from BC = h i.e. AD = h First find M.I. of lamina ABC about BC. M= 1 ah , 2 = where = surface density of lamina B D a B x C C x

M (density = Mass/area) ah 2 B' C' h x Now = BC h a (h x ) BC = = length of strip h a (h x ) x h

Area of strip BC = Mass of strip =

M a ( h x ) x . h ah 2 2M = 2 (h x)x h 2M (hx)x2 x 2 h

M.I. of strip BC about BC =

M.I. of lamina ABC about BC =

h2
0

2M

(h x) x2 dx
h

2M hx 3 x 4 = 2 4 0 h 3 = I=

2M h 4 h 4 2M h 4 + = 2 3 h 12 h2 4 (1)

1 Mh2 6

33

Now we apply this result to general case of finding M.I. about any line in the plane of lamina. D D h2 B G A h1 h3 IG

C Let h1, h2, h3 are length of drawn from corners (or points) A, B, C respectively of ABC. s. t. h1 < h2 < h3 We extend BC to meet a point D on line . We draw a line through A and parallel to . Distances of C and B from are h3 h1, h2 h1 Let M1 is the mass of ACD and M2 is the mass of ABD This and M = M1 M2

1 AD(h 3 h1 ) M1 = 2 M 2 . 1 AD(h h ) 2 1 2 M1 h 3 h 1 = M 2 h 2 h1 M1 M2 M M2 = M = = 1 h 3 h1 h 2 h1 h3 h2 M1 = M(h 3 h1 ) , h3 h 2 M2 = M(h 2 h1 ) h3 h2 (2)

We denote I as the M. I. of ABC about and I as the M.I. of ABC about and IG as the M.I. of ABC about a line parallel to or through centre of mass (G) of ABC. So then I = M.I. of ACD M.I. of ABD

34

= =

1 1 M1(h3 h1)2 M2(h2 h)2 6 6 M (h 3 h1 ) 3 (h 2 h 1 ) 3 6 h3 h2 [using equation (2)]

M (h 3 h 2 ) 2 2 [(h3 h1) + (h2 h1) + (h3 h1) (h2 h1)] 6 (h 3 h 2 ) [ a3 b3 = (a b) (a2 + b2 + ab)]

= =

M 2 2 2 2 [ h 3 + h 1 2 h 1 h 3 + h 2 + h 1 2 h 1h 2 + h 2 h 3 h 1 h 2 h 1 h 3 + h 1 ] 2 6 M 2 2 [3h1 + h 2 + h 3 + h 2 h 3 3h 3 h1 3h1h 2 ] 2 6 (h 1 + h 2 + h 3 ) 3 (3) (4) (5)

Now distance of G from =

h + h2 + h3 h1 and distance of G from = 1 3 Using parallel axes theorem, I = IG + and I = IG + I = M (h1 + h2 + h3)2 9 M (h2 + h3 2h1)2 9

(6) (7)

M 2 (3 h1 + h 2 + h 3 + h 2 h 3 3h1h 3 3h1h 2 + 4h 2c + h1h 3 ) 2 6 M (h2 + h3 2h1)2 9 M M (h1 + h2 + h3)2 (h2 + h3 2h1)2 9 9 (8)

(7) IG = I

Put equation (8) in (6), I = I + =

M 2 2 [3h1 + h 2 + h 3 + h2 h3 3h3h1 3h1h2] 2 6 M M (h2 + h3 2h1)2 + (h1 + h2 + h3)2 9 9

35

M 2 M 2 2 2 [3h 1 + h 2 + h 3 + h 2 h 3 3h 1 h3 3h1h2] + [ h1 + h 2 + h 3 + 2h1h2 2 2 9 6


2 2 + 2h2h3 + 2h3h1 h 2 h 3 4h1 2h 2 h 3 + 4h1h 3 +4h1h2] 2

I =

M 2 2 [ h 1 + h 2 + h 3 + h1 h2 + h 2 h3 + h1 h3 ] 2 6

2 2 2 M h1 + h 2 h 2 + h 3 h 3 + h1 I = + + 3 2 2 2

= M. I. of mass

M placed at mid-point of A and B + 3 M placed at mid-point of B and C + 3

M.I. of M.I. of mass M.I. of mass

M placed at mid-point of C and A. 3 M at the mid-points of sides 3

i.e. which is same as M.I. of equal particles of masses of ABC.

Example 4:- Find equimomental system for a uniform solid cuboid. OR

Show that a uniform solid cuboid of mass M is equimomental with (i) (ii) Solution:(2a,0,2a)6 Masses Masses M M at the mid-points of its edges & at its centre. 24 2 M 2M at its corners & at its centroid. 24 3 z 8 3

7
(2a,2a,2a)

Z 6 (a,a,a) G 9 Y 10 2(0,2a,0) 0 X 1 2 Let length of edge of cuboid = 2a y 1 5 x (2a,0,0) 4 Coordinates of mid-point of edges of cuboid are 1 = (a, 0, 0), 2 = (0, a, 0), 3 = (0, 0, a), 4 = (2a, a, 0), 5 = (a, 2a, 0)

12 7 11

5(0,2a,2a)

36

6 = (0, 2a, a), 7 = )0, a, 2a), 8 = (a, 0, 2a), 9 = (2a, 0, a), 10 = (0, 2a, a) 11 = (a, 2a, 2a), 12 = (2a, a, 2a) Let G be centroid & is the density of cuboid, then M = V = (2a)3 = 8a3, (1) Now we find M.I. and Product of Inertia of cuboid about co-ordinates axes. Therefore, A = M.I. of cuboid about x-axis = =
2 2 2 2 ( y + z )dv = ( y + z )dx dy dz v 0 0 0 2a2a2a

8 2 8 a (8a 3 ) = Ma2 3 3

[using (1)] 8 Ma2 3 8 Ma2 3

Similarly

B = M. I. of cuboid about y-axis = C = M.I. of cuboid about z-axis =

Now D = product of inertia of cuboid w.r.t. pair (oy, oz)


2a 2a2a

D=

yz dx dy dz = (da
0 0 0

) a2

D = Ma2 M are 24

Similarly E= F = Ma2 (i) Now consider a system of particles in which 12 particles each of mass situated at mid-point of edges. i.e. at i (i = 1 to 12) and a particle of mass M M Total mass of this system = 12 + 24 2 = M M + =M 2 2 M at G. 2

The two systems have same mass. Also the centroid of these particles at i and G is the point G itself which is centroid of cuboid. the two systems have same centroid.

37

Let A = M.I. of system of particles at (i, G) about x-axis. = m(y2 + z2) + = M (2a2) 2

M [0 + a2 + a2 + a2 + 4a2 + 5a2 + 5a2 + 4a2 + a2 + 5a2 + 8a2 + 5a2] 24 + M (2a2) 2 M 64 8 (40a 2 ) + Ma 2 = Ma 2 = Ma2 24 24 3

A =

Similarly B = M.I. of system of particle about y-axis = m(z2 + x2) B = 8 Ma2 3 8 Ma2 3

Similarly C =

Now D = M.I. of system of particles w.r.t. (oy, oz) = myz = M M 2 [0 + 0 + 0 + 0a2 + 0 + 2a2 + 2a2 + 0 + 0 + 2a2 + 4a2 + 2a2]+ (a ) 24 2 M M M M (12a 2 ) + a 2 = a 2 + a 2 = Ma 2 24 2 2 2

D =

Similarly

E = F = Ma2

Both the systems have same M.I. and product of inertia referred to co-ordinate axes through O. Using parallel axes theorem, both systems (i.e. cuboid & particles) have identical moments and products of inertia referred to parallel axes through common centroid G. So both the systems have same principal axes and principal M.I. Therefore both the systems are equimomental. (ii) Now let A = M.I. of system of particles at (i & G) about x-axis = M 2 (0 + 4a2 + 4a2 + 4a2 + 8a2 + 4a2 + 8a2) + M (2a2) 24 3

38

= A =

M 4 4 4 (32 Ma2) + Ma2 = Ma2 + Ma2 24 3 3 3 8 Ma2 3 8 Ma2 3

Similarly B = C =

Also D = P.I. of system of particles w.r.t. (oy, oz) axes = = M 2M 2 [0 + 0+ 0 + 0 + 4a2 + 0 + 4a2] + (a ) 24 3 8 2M 2 1 2 Ma 2 + a = Ma 2 + Ma 2 24 3 3 3

D = Ma2

Similarly E = F = Ma2 Both the systems have same M.I. and product of inertia referred to co-ordinate axes through O. Using parallel axes theorem, both systems (i.e. cuboid & particles) have identical moments and products of inertia referred to parallel axes through common centroid G. So both the systems have same principal axes and principal M.I. Therefore both the systems are equimomental.
Self Assessment Questions

1. Find Principal direction at one corner of a rectangular lamina of dimension 2a and 2b. 2. Find equimomental system for a parallelogram or parallelogram is equimomental with particles of masses M/6 at mid-points of sides of ||gm & M at the intersection of diagonals. 3

39

Lesson-3
3.1 Some definations 3.1.1 Generalized Co-ordinates

Generalized

co-ordinates

and

Lagranges Equations

A dynamical system is a system which consists of particles. It may also include rigid bodies. A Rigid body is that body in which distance between two points remains invariant. Considering a system of N particles of masses m1, m2,.mN or mi (1 i N). Let (x, y, z) be the co-ordinates of any particle of the system referred to rectangular axes. Let position of each particle is specified by n independent variables q1, q2,.qn at time t. That is x = x(q1, q2, qn;t) y = y(q1, q2, qn; t) z = z(q1, q2,qn; t) The independent variables qj are called as generalized co-ordinates of the system. Here we use to denote total differentiation w.r.t. time. & q1 = dq1 , dt & qj = dq j dt dq j dt etc. (j = 1,2, 3..n)

& The n quantities q j =

are called generalized velocities.

3.1.2 Holomonic system :- If the n generalized co-ordinate (q1, q2,qn) of a given

dynamical system are such that we can change only one of them say q1 to (q1 + q1) without making any changes in the remaining (n1) co-ordinates, the system is said to be Holonomic otherwise it is said to be Non-Holonomic system.

40

3.1.3 Result :- Let system consisits of N particles of masses mi (1 i N) & qj (j =

1 to n) are generalized coordinates. Let ri be the position vector of particle of mass mi at time t. Then ri = ri (q1, q2,, qn; t) d ri &= Then ri dt ri dq1 ri dq 2 r dq n ri & ri = + + ... + i + t q1 dt q 2 dt q n dt r r & & r & r & ri = q1 i + q 2 i + ... + q n i + i q1 q 2 q n t & & & We regard q1 , q 2 ...q n , t as independent variables. So,
& ri r = i & q j q j

(1)

3.1.4 Virtual displacement:- Suppose the particles of a dynamical system undergo


a small instantaneous displacement independent of time, consistent with the constraint of the system and such that all internal and external forces remain unchanged in magnitude & direction during the displacement.

3.1.5 Virtual Work & Generalised forces:- Consider a dynamical system


consisting of N particles of masses mi (1 i N). Let mi is the mass of ith particle with position vector ri at time t, it undergo a virtual displacement to position ri + ri . Let Fi = External forces acting on mi Fi ' = Internal forces acting on mi Therefore, virtual work done on mi during the displacement ri is (Fi + Fi). ri Total work done on all particles of system is, W =

(Fi + Fi). ri
i =1

41

Fi . ri + Fi. ri
i =1 i =1

where W is called virtual work function. If internal foces do not work in virtual displacement, then so
N

Fi. ri
i =1 N

=0

W =

Fi . ri
i =1

Let Xi, Yi, Zi are the components of Fi and xi, yi, zi are the components of ri i.e. Fi = (X i , Yi , Z i ) and ri = (x i , y i , z i ) Then W =

(X i x i + Yi y i + Zi z i ) .
i =1

If the system is Holonomic ,i.e., the co-

ordinate qj changes to qj + qj without making any change in other (n1) coordinate. Let this virtual displacement take effect & suppose the corresponding work done on the dynamical system to be Qj qj , then Qj qj = Fi . ri
N i =1

Now, if we make similar variations in each of generalized co-ordinate qj, then W =

Q j q j = Fi . ri
j=1 i =1

Here Qj are known as Generalised forces and qj are known as generalised virtual displacements.

3.2 Constraints of Motion:- When the motion of a system is restricted in some


way, constraints are said to have been introduced. Constraints

Holonomic Co-ordinates are related by equations f( r1 , r2 ...rn , t ) = 0

Non-Holonomic Co-ordinates through inequalities

42

Holonomic

Scleronomic 1. Time independent i.e. derivative w.r.t t is zero 2. Independent of & && velocities ( x , y, z)

Rheonomic Time dependent i.e. derivative w.r.t. t is non-zero. Constraints depends explicitly on time

Example of Holonomic constraints


1. 2.

(ri r j ) 2 = constant
f( r1 ,..., rn , t ) = 0

Example of non-Holonomic constraints


Motion of particle on the surface of sphere. Constraints of motion is (r2 a2) 0 where a is radius of sphere.

3.3 Lagranges equations for a Holonomic dynamical system:- Lagranges


equations for a Holonomic dynamical system specified by n-generalised coordinates qj (j = 1, 2, 3.. n) are d T T = Qj, & dt q j q j Consider a dynamical system consisting of N particles. Let mi, ri be the mass, position vector of ith particle at time t and undergoes a virtual displacement to position ri + ri . Let Fi = External fore acting on mi Fi = Internal force acting on mi Then equation of motion of ith particle of mass mi is where T = K.E. of system at time t and Qj = generalized forces.

43

& Fi + Fi = m i ri
The total K.E. of the system is, T= 2 1 N m i ri & 2 i=1

(1)

(2)

Now

d ri n ri & = + qk dt q j t k =1 q k q j d ri ri n r & & = ri = + qk i dt t k =1 q k

(3) r d n & = + qk i dt t k =1 q k

d ri ri n = + q k ri & (3) dt q j t q j k =1 q k q j ri n ri & = + qk q j t k =1 q j q k r ri n & i = + q k q j t q j k =1 q k = n & + qk ri q j t k =1 q k d ri & = ( ri ) = q j dt q j

& [ q k are independent of qj]

& d ri ri = dt q j q j

(4)

Also we know that & ri ri = & q j q j Consider d ri ri d ri & & = ri + ri ri q j dt q j dt q j

(5)

44 ri & ri & & = ri + ri q j q j d ri ri & & ri & & ri & ri = ri & q j dt q j q j = 1 d 2 1 2 & & (r ) ( r ) & i 2 q j i 2 dt q j

[using (4)]

[using (5)]

Multiplying both sides by mi & taking summation over i = 1 to N. d 1 N r 1 & & m i ri i = &q dt q 2 m i ri 2 q 2 m i ri 2 & i=1 j j j i =1
N

d T T r (Fi + Fi) i = q j dt q j q j i =1
N

[using (1) & (2)]

(6)

Also we have the relation, W =

Q j q j = q j = Fi ri = [Fi + Fi]ri
j=1 i =1 i =1

(7)

Since the system is Holonomic, we regard all generalized co-ordinates except qj as constant. Then, (7) gives Qj qj = Qj =
N N

(Fi + Fi)ri
i =1

(8)

r (Fi + Fi) i q j i =1 r (Fi + Fi) i q j i =1


N

Qj =

(9)

from (6) & (9), we get d T T = Q j , j = 1, 2,n & dt q j q j This is a system of n equations known as Lagrange equations.

45

3.4 Example:- Planetary Motion :-

S(fixed)

(m) P(r,) m r2
Initial line

P Planet S Sun(fixed)

Let (r, ) be the polar co-ordinates of P w.r.t. S at time t. Under the action of inverse square law of attraction, force =
& radial velocity = r & transverse velocity = r m r2

Here (r, ) are the generalized co-ordinates of the system and K.E. is T= 1 & & m( r 2 + r 2 2 ) 2 [
& & 2 = r 2 + r 2 2 ]

&& where r, , r, are independent. As the system is Holomonic, the virtual work

function is given by
m W = 2 r + 0 [ W = Q j q j = Q1 q1 + Q 2 q 2 = Q r r + Q 2 ] r

Qr =

m r2

Q = 0 Now
T 1 & & = m( r 2 + r 2 2 ) r r 2 T & = mr 2 r T =0 T & = mr, & r T & = mr 2 &

and

Therefore Lagranges equations are

46

d T T = Qr & dt r r and (1) and d T T = Q & dt d & m & (m r ) = mr 2 = 2 dt r


& & m & mr 2 = r m r2

(1) (2) (3)

m & & (3) m & mr 2 = 2 r r


& r 2 = & r & r2

and (4)

d 2& ( r ) = 0 dt

3.5 Lagrange equation for a conservative system of forces

Suppose that the forces are conservative & the system is specified by the generalized co-ordinate qj (j = 1, 2,.n). So we can find a potential function V(q1, q2,, qn) such that W = V, where V =
n V q W = q j j j=1

V V V q1 + q 2 + ... + q n q1 q 2 q n

Q j q j = q
j=1

V q j j j=1
n

Qj =

V q j

Therefor, Lagranges equation for a conservative holonomic dynamical system becomes

47

d T T V = dt q j q j q j & or Let or

d T (T V ) = 0 & dt q j q j L = T V where L = Lagranges function L = K. E. P. E d T L =0 & dt q j q j

& & & Since V does not depend upon q1 , q 2 ,..., q n V =0 & q j

d L L =0 & dt q j q j Let qj (j = 1, 2,n) be generalized co-ordinate at time t for a Holonomic

3.6 Generalised components of momentum and impulse


& & & dynamical system. Let T = T(q1, q2,., qn, q1 , q 2 ....q n , t ) . Then, the n quantities pj

is defined by pj =
T are called generalized components of momentum. & q j

We know that Lagrange equation is d T T =0 & dt q j q j


T d (p j ) =0 q j dt

Now T =

1 1 1 & & & & m v 2 = m r 2 = m( x 2 + y 2 + z 2 ) 2 2 2


T & = mx & x

Then px =

48

& & Similarly py = m y, pz = mz


For generalized forces Qj (j = 1, 2,.n) for dynamical system, the n quantities Jj defined by
Lt Q j dt = J j (finite) Q j 0 0

when limit exists are called generalised

impulses.

Since W =

Q j q j
j=1 n

W dt = j=1
0
Q j 0 0

q j Q j dt 0
n

Lt

W dt = j=1 J j q j
j=1 n

Lt Q dt q j Q j j 0 0

U =

where U is called impulsive virtual work function.


3.7 Lagranges equation for Impulsive forces

It states that generalized momentum increment is equal to generalized impulsive force associated with each generalized co-ordinate.
Derivation:- We know that Lagranges equation for Holonomic system are

d T T = Qj & dt q j q j T d (p j ) = Qj dt q j (1)

Integrating this equation from t = 0 to t = we get (pj)t= (pj)t=0 = T q j dt + Q j dt 0 0


Let Qj, 0 in such a way that

49

Q j 0 0

Lim Q j dt = J j (finite) (j = 1, 2,., n)

Further as the co-ordinate qj do not change suddenly, dt = 0 0 q j Lt


0

Writing pj = Lt [(pj)t= (pj)t=0],


0

We thus obtain Lagranges equation in impulsive form

pj = Jj, j = 1, 2,.n
3.8 Kinetic energy as a quadratic function of velocities
Let at time t, the position vector of ith particle of mass mi of a Holonomic system is ri , then K.E. is T=

1 N & m i ri 2 2 i=1

(1)

where N is number of particles. Suppose the system to be Holonomic & specified by n generalized co-ordinates qj , then ri = ri (q1, q2,., qn, t) d ri ri ri ri ri & & & & = q1 + q2 + ... + q n + (2) ri = q1 q 2 q n t dt From (1) & (2),

2 ri r r r 1 N & & i + ... + q n i + i & q1 T = mi + q2 q n t q 2 2 i=1 q1 2 ri ri ri 1 N & & & = m i q1 + ... + q n + q2 q n q 2 2 i=1 q1 2 N r r r 1 N r & & + m i i + m i i q1 i + ... + q n i q n t q1 2 i=1 t i =1 T= 1 2 & & & && [(a 11q1 + a 22 q 2 + ... + a nn q 2 + 2a 12 q1q 2 + ....) 2 n 2 & & & + 2(a1 q1 + a 2 q 2 + .... + a n q n ) + a ] (3)

50

ri ri where ars = asr = m i q q , s r r r s i =1 N ri ri ar = m i q t r i =1


N

2 ri a = mi t i =1
N

equation (3) shows that T is a quadratic function of the generalized velocities. Special Case :- When time t is explicitly absent, then ri = ri (q1, q2,.qn) d ri ri ri ri & & & & + ... + q n + q2 = q1 ri = q 2 q1 dt q n ri and =0 t From (3), we get T= = 1 2 & & & && [a 11q1 + a 22 q 2 + ... + a nn q 2 + 2a 12 q1q 2 + ....] 2 n 2 1 n n && a rs q r q s 2 s=1 r =1

Thus the K.E. assumes the form of a Homogeneous quadratic function of the & & & generalized velocities q1 , q 2 ...q n . In this case, using Eulers theorem for Homogeneous functions & q1 T T T & & + q2 + ... + q n = 2T & & & q1 q 2 q n

& & & q1p1 + q 2 p 2 + ... + q n p n = 2T

3.9 Donkins Theorem :-Let a function F(u1, u2,., un) have explicit dependence
on n independent variables u1, u2un. Let the function F be transformed to another function G = G(v1, v2 vn) expressed in terms of a new set of n independent variables v1, v2, vn where these new variables are connected to the old variables by a given set of relation

51

Vi =

F , i = 1, 2..n u i
n

(1)

& the form of G is given by G(v1, v2. vn) =

u i vi
i =1

F(u1, u2.un)

(2)

then the variables u1, u2 un satisfy the dual transformation ui = & G v i (3)

F(u1, u2 un) =

u i vi
i =1

G(v1, v2.vn)

This transformation between function F & G and the variables ui & vi is called Legendres dual transformation.

Proof : Since G is given by


G(v1, v2,., vn) =

u k vk
k =1

F(u1, u2,., un)

Then

G n = u k v k F(u1 , u 2 ...u n ) v i v i k =1 =
n n u k V F u k vk + u k k v Vi k =1 u k dv i i k =1 i =1 n

n n v H u k G = k v k + u k ki v i k =1 v i k =1 u k v i

n u k F u k vk + ui v i k =1 k =1 u k v i n

vk u
k =1 i

F
k

+ ui

F u k k =1 u k v i
n

F (1) v k = u k

= ui G = ui v i

52

3.10 Extension of Legenders dual transformation


Further suppose that there is a another set of m independent variables 1, 2n present in both F & G. F = F(u1, u2.un, 1, 2. m) G = G(v1, v2 vn, 1, 2.m) then there should be some extra condition for Legendres dual transformation to be satisfied. These conditions are F G = , j = 1, 2.m L.H.S. j j Consider G = G(v1, v2vn, 1, 2 m) = From L.H.S. G = From R.H.S., G =
m G G v i + j v i i =1 j=1 j n

u i v i F(u1, u2un, 1, 2 m) R.H.S.


i =1

(1)

u i v i + v i u i
i =1 i =1

m F F j u i u i j i =1 j=1

(2)

Equating (1) & (2),

v
i =1

G
i

v i +

n n n m G F F j = u i u i + v i u i u i j j=1 j i =1 i =1 i =1 u i j=1 j

vi = ui =

F are satisfied provided u i G v i

and

G F = j j

53

Lesson-4
4.1 Introduction

Hamiltons Equations of Motion

So far we have discussed about Lagrangian formulation and its application. In this lesson, we assume the formal development of mechanics turning our attention to an alternative statement of the structure of the theory known as Hamiltons formulation. In Lagrangian formulation, the independent variables are qi and q i , whereas in Hamiltonian formulation, the independent variables are the generalized coordinates qi and the generalized momenta pi
.

4.2 Energy equation for conservative fields


Prove that for a dynamical system T + V = constant where T = K.E. V = P.E. or ordinary potential

Proof : Here V = V(q1, q2,, qn)


& & & T = T(q1, q2 . Qn, q1 , q 2 ...q n ) & & & L = T V = L(q1, q2qn, q1 , q 2 ,...., q n ) If Lagrangian function L of the system does not explicitly depend upon time t, then L =0 t i.e. & L = L(qj, q j ) for j = 1, 2,n

The total time derivative of L is


n dL n L L & & & = qj + q & j dt j=1 q j j=1 q j

(I)

We know that the Lagranges equation is given by d L L =0 dt q j q j & (I) (II)

dL n d L n L + & & & q = q j & & j dt j=1 dt q j j=1 q j

[using (ii)]

54

& dt q j q &
j=1

L
j

L dn & L = 0 q j & dt j=1 q j dH =0 dt & where H = q j


j=1 n

(1) L L & q j

is a function called Hamiltonian H= & q j p j L


j=1 n

(A) L = p j = generalized component of momentum] & q j

[ Integrating (1),

& q j q L = constant &


j=1 n j

(2)

Now

& q j
j=1

n T L & = q j & & q j j=1 q j

1 n N & & q j q m i ri 2 2 j=1 & i=1 j


n

2 1 N & T = m i ri 2 i=1

N & = q j m i i=1 j=1


n

ri & & ri q j & ri & ri = & q j q j

N r & & = q j m i ri i q j i=1 j=1 =


N

n N r && m i ri i q j = m i ri ri & q & j=1 j i=1 i =1 (3)

= 2T

55

from (2) & (3), 2T L = constant. 2T (TV) = constant T + V = constant. [ L = TV]

Also from (A), H = T + V = constant. Total energy T + V =H, when time t is explicitly absent.

4.3 Generalised potential


For conservative forces, Potential function V = V(q1, q2qn), therefore W = V V V = x x i = q q j j i Also W = Qj qj where Qj are generalized forces.

Q j q j = q
Qj = V q j

V q j j

4.4 Cyclic or Ignorable co-ordinates


Lagrangian L=TV If Lagrangian does not contain a co-ordinate explicitly, then that co-ordinate is called Ignorable or cyclic co-ordinate. Let & & & L = L(q1, q2qn, q1 , q 2 ,...q n , t ) L =0 q k Lagranges equation (equation of motion) corresponding to qk becomes d L dt q k & 0 = 0 L = constant = pk & q k

Let qk is absent in L, then

56

4.5 Hamiltonian and Hamiltonian variables:- In Lagrangian formulation,


independent variable are generalized co-ordinates and time. Also generalised velocities appear explicitly in the formulation. & L(qk, q k , t,)

& Like this Lagrangian L(qj, q j , t), a new function is Hamiltonian H which is function of generalized co-ordinates, generalized momenta and time ,i.e., H(qj, pj, t), where pj = L & q j

Also we have shown that H=

& pj q j L
j

This quantity is also known as Hamiltonian. The independent variables q1, q2,qn , p1, p2pn, t are known as Hamiltonian variables.
4.6 Hamiltons Canonical equations of motion

Lagranges equations of motion are d L L =0, & dt q j q j Now H = H(qj, pj, t) H= & & p j q j L (q j , q j , t )
j=1 n

j = 1, 2,.n (1) (2)

The differential of H from (1), dH = from (2) dH = & & [p j dq j + q jdp j ] q


j=1 n n

H
j

dq j +

H H dp j + dt p j t

(3)

L
j

dq j

& q dq j &
j

L dt t (4)

dH =

n n L L & & dq j + q j dp j d qj q j j=1 & j=1 j=1 q j

L p j = & q j

57

From Lagranges equation, & pj = L q j

d L (p j ) = dt q j (5)

Using (4) & (5), we get dH = & & q j dp j p j dq j t


j=1 n

dt

(6)

Comparing equation (3) & (6), we get H H & & = q j, p j = p j q j and H L = t t where j = 1, 2,n (7) (8)

The equation (7) is called Hamiltonians canonical equations of motion or Hamiltons equations.
Result:- To show that if a given co-ordinate is cyclic in Lagrangian L, then it will

also be cyclic in Hamiltonian H. If L is not containing qk ,i.e., qk is cyclic, then & then p k = 0 pk = constant From equation (1), H(qj, pj, t) H(q1, q2,.., qk1, qk+1,., qn, p1, p2,. pk1, pk+1.pn, t) If H is not containing t ,i.e., H = H(qj, pj) then H dH H & & = qj + pj q j p j dt L =0 q k

Using equation (7) or Hamiltons equation dH H H H H =0 = dt q j p j p j q j

58

dH =0 dt

H = constant.

If the equation of transformation are not depending explicitly on time & if P.E. is velocity independent, then H = E (total energy) Which can also be seen from the expression as given under ri = ri ( q1, q2,., qn) P.E., V = V(q1, q2,., qn) K.E., T = Now 1 N & m i ri 2 2 i=1

n r & & ri = i q j j=1 q j


2 n ri 1 N & T = mi qj 2 i=1 j=1 q j

& & & = (quadratic function of q1 , q 2 ,...., q n ) Therefore by Eulers theorem for Homogeneous function, we have & q j q = 2T &
j

H=

& & p j q j L = q j q L &


j

& q j q &
j

L = 2T L

H = 2T (T V) = T + V = E H=E

Example:- Write the Hamiltonian & Hamiltons equation of motion for a particle in

central force field (planetary motion).


Solution : Let (r, ) be the polar co-ordinates of a particle of mass m at any

instant of time t. Now L = TV(r) As L= 1 & & m[r 2 + r 2 2 ] V(r) 2

where V(r) = P.E. (1) P(r,) (m) & r O X

qj = r,

59

& && q j = r, pj = pr, p Now pr = H= L L & & = mr, p = = mr 2 & & r & & & & & p j q j L = p r r + p 2 m( r 2 + r 2 2 ) + V ( r ) 1 (2)

& 1 & 1 & & = m r 2 + mr 2 2 m(r 2 ) mr 2 2 r + V(r) 2 2 = 1 & & m( r 2 + r 2 2 ) + V( r ) 2 1 pr m (3)

H=T+V

& from (2), r =

& 1 p = mr 2
2 2 1 p r 2 p Then H = m + r 2 + V(r) 2 m mr

2 1 2 p H= p r + 2 + V(r), which is required Hamiltonian 2m r

Hamiltons equations of motion are, & qj = H , p j & pj = H q j

& The two equations for q j are & r= H p r & = = qr p r m

& H = p = q & Similarly = p mr 2 & and two equations for p j are,

60

& pr = and & p =

2 H p V(r ) = 3 r r mr

H =0

p = constant

4.7 Rouths equations

Routh proposed for taking some of Lagrangian variables and some of Hamiltonian variables. The Routh variables are the quantities & t, qj, q, q j , p j = 1,2,.k = k + 1, k + 2, .n k is arbitrary fixed number less than n. Rouths procedure involves cyclic and noncyclic co-ordinates. Suppose co-ordinates q1, q2,., qk (k < n) are cyclic (or Ignorable). Then we want to find a function R, called Routhian function such that it does not contain generalized velocities corresponding to cyclic co-ordinates. & & & L = L(q1, q2.qn, q1 , q 2 ,..., q n , t ) If q1, q2.qk are cyclic, then & & & L(qk+1,., qn, q1 , q 2 ,....q n , t ) so that dL =
n L L L & dt dq j + dq j + q & t j j= k +1 j=1 q j n

k n n d L L dq j = L dq j + L dq j + L dt & & j=k +1 q j & & q j q j t j=1 k +1

(1)

& & & Routhian function R, in which velocities q1 , q 2 ....q k corresponding to ignorable coordinate q1, q2,., qk are eliminated, can be written as & & R = R(qk+1, qk+2,., qn, q k +1 ,....q n , t ) so that

61

dR =

n R R R & dq j + dq j + dt & t j= k +1 q j j= k +1 q j

(2)

Further we can also define Routhian function as R=L & q j p j


j=1 k

We want to remove
k

q j
j=1

or

& q j
j=1 k

from L to get R.

dR = dL

& & q j dp j p j dq j
j=1 k j=1 k L & & dq j q j dpj q j j=1 & j=1

= dL
n

[using (1)]

dR =

n k L L L & & dq j + dq j + dt q j dp j q q j t j j= k +1 j= k +1 j=1

(3)

Comparing (2) & (3) by equating the coefficients of varied quantities as they are independent, we get L R = , q j q j L R = , t t d L
j

L R = & & q j q j j = k + 1, k + 2,.n

(4)

Put (4) in Lagrangians equations,

dt q q &
j=1

L =0 j

j = 1, 2,.n

we get,

j= k +1

dt q q &

d R

R =0 j , j = k + 1,.n

or

d R R =0 & dt q j q j

62

These are (nk) 2nd order equations known as Rouths equations.


4.8 Generalised potential

When the system is not conservative. Let U is Generalised potential, say it depends

& on generalised velocities ( q j ) i.e. we consider the case when in place of ordinary & potential V (qj, t), there exits a generalised point U(qj, t, q j ) in terms of which the
generalised forces Qj are defined by Qj = d , U U dt q j q j & j = 1,2,n

[ L = TV for conservative system, system]

L = TU for non-conservative

Here U is called generalised potential or velocity dependent potential. Here Lagrange equations are d T T d U U = Qj = & & dt q j q j dt q j q j

d (T U ) (T U ) = 0 dt q j q j & d L L =0 & dt q j q j [ L = T U for non-conservative system]

4.9 Poissons Bracket

Let A and B are two arbitrary function of a set of canonical variables (or conjugate variables) q1, q2,., qn, p1, p2.pn , then Poissons Bracket of A & B is defined as [A, B]q,p = A B A B p j q j j p j

q
j

If F is a dynamical variable, i.e., F = F(qj, pj, t), then

63

F F F dF dF & & = (q j , p j , t ) = qj + pj + p j t dt dt j q j Using Hamiltons canonical equations, & qj = H , p j & pj = H q j

(1)

from (1),

F H F H F dF + = q j p j p j q j t dt

F dF = [F, H]q ,p + t dt F = 0, t

If F is not depending explicitly on t, then

so

F H F H dF = q j p j p j q j dt = [F, H]q,p

4.9 Properties

I. II. III. IV.

[X, Y]q,p = [Y, X]q,p [X, X] = 0 [X, Y+Z] = [X, Y] + [X, Z] [X, YZ] = Y[X, Z] + Z[X, Y] X Y X Y p j p j q j j

Solution :- I. By definition [X, Y]q,p =

q
j

[Y, X]q, p =

q
j

Y X Y X p j q j j p j

X Y X Y = p j q j j q j p j [Y, X]q,p = [X, Y]

64

II.

[X, X]q,p =

q
j

X X X X =0 p j q j j p j

Also

[X, C]q,p =

q
j j

X C X C =0 p j q j j p j

III.

[X, Y + Z]q,p =

q
j

X (Y + Z) X (Y + Z) p j p j q j j X Y Z X Y Z + + j p j p j p j q j q j

q q
j

[X, Y + Z]q,p =

X Y X Y + p j q j j p j

q
j

X Z X Z p j q j j p j

= [X, Y] + [X, Z] IV. [X, YZ]q,p =

q
j

X (YZ) X (YZ) p j q j j p j

X Z Z Y X Y Z Y = +Y +Z q j p j p j q j q j p j X Z X Z + Z X Y X Y = 4 j q j p j p j q j j q j p j p j q j = Y[X, Z] + Z[X, Y] Also (i) (ii) (iii) [qi, qj]q,p = 0 [pi, pj]q,p = 0 1, [qi, pj]q,p = ij = 0, i= j i j

Solution:(i) [qi, qj]q,p =

q i
k

q q j q i q j k p k p k q k

(1)

65

Because qi or qj is not function of pk q i = 0, p k q j p k =0

(1) [qi, qj]q,p = 0. (ii) [pi, pj]q,p =

q i
k

p p j p i p j k p k p k q k p j q k

As pi, pj is not a function of qk (iii) p i = 0, q k [pi, pj]q,p= 0 Now [qi, pj]q,p = =0

q i
k

q p j q i p j k p k p k q k

= =

q i
k k

q p j q p j 0 = i k p k k q k p k
k

ik jk = ij = ij
i= j i j

1, [qi, pj]q,p = ij = 0,

Some other properties:-

If [, ] be the Poisson Bracket of & , then (1) (2) [, ] = , + , t t t d d d [, ] = , + , dt dt dt [, ] = t t i q i p i p i q i =

Solution:- (1)

t q
i

i p i p i q i

66

t q
i

+ i p i q i t p i

+ t p i q i p i t q i

q
i

p q p t + q p i i i i i t i

q
i

+ i t p i q i p i t q i t p i q i p i t i t p i q i p i t +

q
i

q
i

i p i t p i q i t

[, ] = , + t t

, t .

(2) Similarly, we can prove d d d [, ] = , + , dt dt dt


4.9.1 Hamiltons equations of motion in Poissons Bracket:-

If then

H Hamiltonian [q, H]q,p = [p, H]q,p = H & =q p H & =p q H & =q p q j H q j H i p i p i q i

From Hamiltons equations, H & = p, q [qj, H] =

q
i

67

[qj, H]=

ji p
i

H
i

q j = 0 p i

= Also But

H p j

& [pj, H] = p j [pj, H] = 0 pj = 0 pj = constant

4.10 Jacobis Identity on Poisson Brackets (Poissons Identity):-

If X, Y, Z are function of q & p only, then [X, [Y, Z]] + [Y, [Z, X]] + [Z, [X, Y]] = 0
Proof : [X, [Y, Z]] + [Y, [Z, X]] = [X, [Y, Z]] [Y, [X, Z]]

Y Z Y Z = X, q j p j p j q j j X Z X Z Y, p j q j j q j p j Let (1)

q
j j

Y Z = E, j p j

p
j

Y Z =F j q j

X Z = G, j p j

p
j

X Z =H j q j

(1) [X, [Y, Z]] [Y, [X, Z]] = [X, EF] [Y, GH] = [X, E] [X, F] [Y, G] + [Y, H] Let E= (2)

q
j

Y Z Y Z = j q j j p j j p j

E = E1 E2 RHS of (2) becomes

Similarly F = F1 F2, G = G1 G2, H = H1 H2

68

[X, E] [X, F][Y, G]+[Y, H] = [X, E1 E2]+[Y,H1 H2][X,F1F2][Y, G1G2] = [X, E1] E2 +[X, E2] E1 [X, F1] F2 [X, F2] F1 [Y, G1] G2 [Y, G2] G1 + [Y, H1] H2 + [Y, H2] H1 Y Z X, Y Z RHS of (2) is = X, j q j p j j p j q j X Z + Y, X Z Y, j q j p j j p j q j Using properties [X, E1 E2] = [X, E1] E2 + [X, E2] E, Y Z Z Y + X, = X, q j p j p j q j Y Z Z Y X, X, p j q j q j p j X Z Z X Y, Y, q j q j p j q j X Z Z X + Y, + Y, p j q j q j p j =

q
j

Z X Y Z X Y + , Y + X, , Y X, p j p j p j q j q j j Y Z Y Z X Z X Z X, X, Y, + Y, j p j p j q j q j p j p j q j (3)

q
j

Using the identity, X Y [ X, Y ] = , Y + X, t t t Then, we find that R.H.S. of equation (3) reduces to

69

q
j

Z Z [X, Y] [ X, Y ] + p j q j j p j

+ 0 (All other terms are cancelled) Z [X, Y] Z [X, Y] = p j p j q j j q j = [Z, [X, Y]] or [X, [Y, Z]] + [Y, [Z, X]] = [Z, [X, Y]] [X, [Y, Z]] + [Y, [Z, X]] + [Z, [X, Y]] = 0

Particular Case

Let Z = H, then [X, [Y, H]] + [Y, [H, X]] + [Y, [X, Y]] = 0 Suppose X & Y both are constants of motion, then [X, H] = 0, [Y, H] = 0 Then Jacobis identity gives [H, [X, Y]] = 0 [X, Y] is also a constant of Motion. Hence poissons Bracket of two constants of Motion is itself a constant of Motion.
4.11 Poissons Theorem

The total time rate of evolution of any dynamical variable F(p, q, t) is given by dF F = + [F, H] dt t
Solution :

F F F dF & & + (p, q, t ) = qj + pj t p j dt j q j = F H F H F + t p j q j j q j p j

70

F dF = + [F, H] dt t If F is constant of motion, then Then by Poissons theorem, F + [F, H] = 0 t Further if F does not contain time explicitly, then [F, H] = 0 F =0 t dF = 0. dt

This is the requirement condition for a dynamical variable to be a constant of motion.


4.12 Jacobi-Poisson Theorem :- (or Poissons Second theorem)

If u and v are any two constants of motion of any given Holonomic dynamical system, then their Poisson bracket [u, v] is also a constant of motion.
Proof:- We consider

d [u, v] = [u, v] + [[u, v], H] dt t

(1)

using the following results, u v [ u , v] = , v + u , t t t [u, [v, w]] + [v, [w, u]] + [w, [u, v]] = 0 (1) d u v [u , v] = , v + u, + [[u , v], H] dt t t (2) (3) (4)

Put w = H in (3), we get [H, [u, v]] = [u, [v, H]] [v, [H, u]] [[v, H], u] [[H, u], v] = [[u, v], H] (5)

from (4) & (5), we get d u v [u , v] = , v + u, [[v, H], u] [[H, u], v] dt t t

71

u v = , v + u, + [u, (v, H]] + [[v, H], v] t t u v = + [u , H], v + u, + [ v, H] t t Because d du dv [ u , v] = , v + u , dt dt dt du dv and both are zero as u & v were constants of motion. dt dt d [u, v] = 0 dt (6)

(6)

The Poisson bracket [u, v] is also a constant of motion.

4.13 Derivation of Hamiltons Principle from Lagranges equation:- We know

that Lagranges equations are d L L =0 & dt q j q j Now & L dt = L(q j q j )dt


t1 t1 t2 t2 L L & L dt = q j + q j dt q j & q j t1 t1 j t2 t2

(1)

t2 L L & q j dt + q dt = & j q j t1 t1 q j t2

K L = q j dt + q & j q j j q j t
1

t2

t2

t1

t2 t1

dt q j q j dt (2) & j

d L

Since, there is no coordinate variation at the end points. q j


t1

= q j

t2

=0

72

So (2)

t2 t2 L d L qj dt L dt = & j q j dt q j t1 t1

L dt = 0 q j dt
t1 t1

t2

t2

[Using (1)]

=0
4.14 Derivation of Lagranges equations from Hamiltons principle
t2 t1

We are given L dt = 0 As qj are arbitrary & independent of each other, So its coefficients should be zero separately. So we have L
j

q
j

d L = 0 & dt q j for j = 1, 2,.n

d L L =0 & dt q j q j

4.15 Principle of Least action

Action of a dynamical system over an interval t1 < t < t2 is


t2

A=

t1

2T dt

where T = K.E. This principle states that the variation of action along the actual path between given time interval is least, i.e., 2T dt = 0
t1 t2

(1)

Now we know that T + V = E (constant) V = P.E. and L = TV

73

By Hamiltons principle, we have


t2 t1

L dt = 0 or

t2 t1

(T V)dt = 0

t2 t1

(T E + T) dt = 0 [(2T) E] dt = 0 (2T) dt = 0
t2 t1

t2 t1

t2 t1

[using E = constant

E = 0]

2T dt = 0

4.16 Distinction between Hamiltons Principle and Principle of least action:-

Hamiltons principle S = 0 is applicable when the time interval (t2 t1) in passing from one configuration to the other is prescribed whereas the principle of least action i.e. A = 0 is applicable when the total energy of system in passing from one configuration to other is prescribed and the time interval is in no way restricted. This is the essential distinction between two principles.
4.17 PoincareCartan Integral Invariant :- We derive formula for W in the

general case when the initial and terminal instant of time, just like initial & terminal co-ordinates are not fixed but are functions of a parameter .
t2

W() =
let

t1

& L [t1 qj(t, ), q j (t, )] dt

t1 = t1(), t2 = t2()

& & q j = q j () at t = t1
q 2 = q 2 () at t = t2 j j

74

Now

W = L dt = L2 t2 L1 t1 +
t1

t2

t2 t1

q j q j + q j q j & j

dt

Integrating by parts Then W = L2 t2 +

p 2j [q j ]t =t 2 L1 t1 pj[q j ]t =t1
j j t2

+ Now qj = qj(t, )

L d L q j dt q j q j dt & t1 j

(1)

q j ( t , ) & [q j ]t = t1 = t = t1 [q j ]t = t 2 = q j ( t , ) t =t 2 q 2 = q 2 q[t(), ] j j (2)

&

On the other hand, for the variation of terminal co-ordinates

q j & ( t , ) q 2 = q 2 t 2 + j j t=t2 & q 2 = [q j ]t = t 2 + q 2 t 2 j j & [ q j ]t = t 2 = q 2 q 2 t 2 j j [using (2)] (3) (4)

& Similarly [q j ]t = t1 = qj q j t1
Put (3) & (4) in (1), we get W = L2 t2 + & p 2j (q 2j q 2j t 2 ) L1 t1
j

t2 L d L qjdt & pj (qj q jt1 ) + & dt q j j q j j t1 2

n t 2 L d L qj dt W = p j q j H t + & j=1 1 t1 j q j dt q j

(5)

75

n where p j q j H t = p 2 q 2 H 2 t 2 j j j=1 1 j & Now we know that H = L p j q j


j

pj qj
j

+ H1 t1

and

H1 = L1 H2 = L2

& pj qj
j

& p 2j q 2j
j

In the special case for any , the path is extremum, the integral on R.H.S. of equation (5) is equal to zero and formula for variation of W takes the form
n W = p j q j H t j=1 1 Integrating, we get n W = p j q j Ht dt j=1 which is known as Poincare Cartan Integral Invariant.
2

(6)

4.18 Whittakers Equations :- We consider a generalised conservative system i.e.


an arbitrary system for which the function H is not explicitly dependent on time. For it, we have H(qj, pj) = E0 (constant) where j = 1, 2,.n (2n dimensional phase space in which qj, pj are coordinates) Then basic integral invariant I will becomes I= I= (1)

( p j q j H t )
p j q j j=1
n

[ for a conservative system, t = 0]

(2)

solving (1) for one of the momenta, for example p1. p1 = k1(q1, q2,., qn, p2pn, E0) (3)

76

Put the expression obtained in (2) in place of p1, we get n I = p j q j + p1 q1 j=2 n = p j q j k1 q1 j=2 (4)

But this integral invariant (4) again has the form of Poincare Cartan integral if it is assumed that the basic co-ordinate and momenta are quantities qj & pj (j = 2, 3...n) & variable q1 plays the role of time variable (and in place of H, we have function k1). Therefore the motion of a generalised conservative system should satisfy the following Hamiltons system of differential equations (2n 2). & qj = dq j dt = k1 k1 dp j ; = p j q j dq1 j = 2, 3. (5)

The equations (5) were obtaioned by Whittaker and are known as Whittakers equations.

4.19 Jacobis equations :Integrating the system of Whittakers equations, we find qj & pj (j = 2, 3,.n) as functions of variables q1 and (2n 2) arbitrary constant C1, C2. C2n2. Moreover, the integrals of Whittakers equations will contain an arbitrary constant E0 ,i.e., they will be of the form qj = j (q1, E0, C1, C2. C2n2) pj = j(q1, E0, C1, C2. C2n2) Putting expression (6) in (3), we find p1 = 1 (q1, E0, C1, C2.. C2n2) From equation (7) (j = 2, 3,.n) (6)

dq1 H = dt p1

dt =

dq1 H p 1

77

t=

H1 + C2n1
p 1

dq

(8)

H where all the variables in partial derivative p are expressed in terms of q1 with 1 the help of (6) & (7). The Hamiltonian system of Whittakers equations (5) may be replaced by an equivalent system of equations of the Lagrangian type: d P P =0, dq1 qj q j j = 2, 3,.n (9)

These are (n 1) second order equations where q j = dq j dq1

and the function P(analogous to Lagrangian function) is connected with the function K1 (analogous of Hamiltonian function) by the equation p = p(q1, q2.qn, q 2 ,...q n ) P=

p j qj K1
j= 2

(10) dq 2 , dq1

The momenta pj must be replaced by their expression in temrs of q '2 = q n = dq n dq1

which may be obtained from first (n1) equation (5). From (3) & (10), P= P=
n

p j qj + p1 =
j= 2

1 n & p q & i i q1 i=1 (11)

1 (L + H) & q1

For conservative system, L = T V, H = T + V

78

L + H = 2T 2T & q1 1 n && a ik q i q k 2 i,k =1 (13) (12)

Then P = & K.E., T =

2 & = q1 G (q1 , q 2 .....q n , q2 ...qn )

where G(q1, q2.qn, q 2 ...qn ...qn ) = From (1) & (13), we find H=E
2 & T = G q1

1 2

i ,k =1

a ik qi qk

(14)

& q1 = and & q1 =

T HV = G G EV G (15)

2 & 2T 2q1 G & P= = = 2G q 1 & & q1 q1

P = 2G

EV G

[from (15)] (16)

= 2 G (E V)

Differential equation (9) in which function P is of the form (16) and which belong to ordinary conservative system are called Jacobis equations.

4.20 Theorem of Lee-Hwa-Chung


If I =

[A i (qk, pk, t) qi + Bi(t, qk, pk)pi] i =1

is a universal relative integral invariant, then I = c I, where c is a constant and I1 is Poincare integral.

79

For

n = 1, I = (Aq + Bp) I = c pq = cI1


n I1 = [p i q i ] i =1

and I1 = pq Ht from Poincare Cartan integral.

80

Lesson-5

Canonical Transformations

5.1 Introduction:-The Hamiltonian formulation if applied in a straightforward


way, usually does not decrease the difficulty of solving any given problem in mechanics; we get almost the same differential equations as are provided by the Lagrangian procedure. The advantages of the Hamiltanian formation lie not in its use as a calculation tool, but rather in the deeper insight it affords into the formal structure of mechanics. We first derive a specific procedure for tranforming one set of variables into some other set which may be more suitable. In dealing with a given dynamical system defined physically, we are free to choose whatever generalised coordinates we like. The general dynamical theory is invariant under transformations qiQi, by which we understand a set of n variable expressing one set of n generalised co-ordinates qi in term of another set Qi. Invariant means that any general statement in dynamical theory is true no matter which system of coordinates is used. In Hamiltonian formulation, we can make a transformation of the independent coordinates qi , pi to a new set Qi, Pi with equations of transformation Qi = Qi (q,p,t), Pi = Pi (q,p,t) Here we will be taking transformations which in the new coordinates Q, P are canonical.

5.2 Point transformation


Qj = Qj(qj, t) Transformation of configuration space is known as point transformation.

5.3 Canonical transformation


old variables new set of variable qj, pj Qj, Pj Here Qj = Qj (qj, pj, t) Pj = Pj(qj, pj, t) If the transformation are such that the Hamiltons canonical equations (1)

81

& qj =

H H , pj = p j q j

Preserve their form in the new variables i.e. & K , Qj = Pj & K Pj = Q j

K being Hamiltonian in the new variable, then transformations are said to be

canonical transformation.
Also if & p j q j L in old variable, then in new variable, & K = P j Q j L' H= where L = new Lagrangian Now d L' L' =0 & dt Q j Q j

i.e. Lagranges equations are covariant w.r.t. point transformation & if we define Pj as Pj = & L' (Q j , Q j ) & Q
j

& Qj = and & Pj =

K (Q j Pj , t ) Pj K (Q j , Pj , t ) Q j
t2 t1

Hamiltons principle in old variable, L dt = 0


t2 t1

& [ p j q j H(q, p, t )]dt = 0

(2)

and in new variable,

82

t2 t1

[ P

& Q j k (Q, P, t ) dt = 0

(3)

Let

t2 t1

& & {( p jq j H ) ( Pj Q j k )}dt = 0

(4)

F = F(q, p, t)
t2 t1

d 2 F(q, p, t ) = [F(q, p, t )]tt1 dt = F


F F = p j q j + p j q j F q j = q j
t1 t2

t1 t2

t1

F p j + p j

=0
t1

[Since the variation in qj and pj at end point vanish] (4) & pj q j H t1


t2

& ) ( Pj Q j K ) dF dt = 0 dt (5)

& & ( p jq j H ) ( Pj Q j K ) = dF dt

In (5), F is considered to be function of (4n + 1) variables i.e. qj, pj, Qj, Pj, t. But two sets of variables are connected by 2n transformation equation (1) & thus out of 4n variables besides t, only 2n are independent. Thus F can be fuinction of F1(q, Q, t), F2 (q, P, t), F3(p, Q, t), or F4(p, P, t) So transformation relation can be derived by the knowledge of function F. Therefore it is known as Generating Function. Let F1 = F1(q, Q, t) & & p j q j H = Pj Q j k + dF1 (q, Q, t ) dt (6)

83

and

F dF1 F & F & (p, Q, t ) = 1 q j + 1 Q j + 1 q j dt Q j t F F F1 t

(7)

from (6) & (7), we get & & & & p j q j H = Pj Q j k + q1 q j + Q1 Q j +


j j j j j

or

& & q1 p j q j + Pj + Q1 Q j + H K +
j

F
j

F1 =0 t

(8)

Since qj and Qj are to be considered as independent variables, equation (8) holds if the coefficients of qj and Qj separately vanish i.e. F1 (q, Q, t) = pj q j and and Pj = F1 (q, Q, t ) Q j F1 (q, Q, t ) t (9) (10) (11)

K=H+

equation (11) gives relation between old and new Hamiltonian. Solving (9), we an find Qj = Qj(qj, pj, t) which when put in (10) gives Pj = Pj(qj, pj, t) equation (12) are desired canonical transformation. (12)

5.4 HamiltonJacobi Equation :-If the new Hamiltonian vanish, i.e., K = 0, then
Qj = j (constant) Pj = j (constant) Also H+ F1 =K=0 t F1 =0 t

H(qj, pj, t) +

Using (11),

F F H q j, 1 , t + 1 = 0 q j t

84

This partial differential equation together with equation (9) is known as HamiltonJacobi Equation. Generating function is also called characteristic function Let F1 is replaced by S, then S S S S , ,..., ,t = 0 , + H q1 , q 2 ,..., q n , t q1 q 2 q n (9)

The solution S to above equation is called Hamilton principle function or characteristic function. Equation (9) is first order non-linear partial differential equation in (n + 1) independent variables (t, q1, q2,,qn) and one dependent variables S. So Therefore, there will be (n + 1) arbitrary constants out of which one would be simply an additive constant and remaining n arbitrary constants may appear as arguments of S so that complete solution has the form S = S(q, t, ) + A where i = 1, 2,, n are n constants and A is additive constant. Jacobi proved a theorem known as Jacobic theorem that the system would volve in such a way that the derivatives of S w.r.t. s remain constant in time and we write S = i (constant) (i = 1, 2, ., n) i i = Ist integrals of motion i = IInd integrals of motion (10)

5.5 Statement of Jacobis Theorem


If S(t, qi, i) is some complete integral fo Hamilton Jacobi equation (9) then the final equations of motion of a holonomic system with the given function. H may be written in the form S S = p i and = i q i i where i, i are arbitrary constants

Proof: Given the complete integral for S given by (10), we wish to prove

85

S = i i Consider d S dt i S S = & t + q q j j i i = S S q j & + i t i q j i 2S S & q [using (9)] = p j + H t , q j , q j q j i j

d S dt i

H q j H 2S 2S & = + qj q S i q j i q j j i q j q j i =0

Since qjs and is are independent, we get

2S d S H & = + q j = 0, Using Hamiltons equation of motion dt i p j i q j H & q j = p j

d S dt i

=0 S = constant = i i (i = 1, 2,, n)

Remark :Consider total time derivative of S(qj, j, t)


S S dS S & & = qj + j + t dt q j j

86

But we know that j = 0 since j are constant. Also S = H , we have t dS & = pj qj H = L dt S = L dt + constant

S S = 0 gives = pj & H + t q j

The expression differs from Hamiltons principle in a constant show that this time integral is ofindefinite form. Thus the same integral when indefinite form shapes the Hamiltons principle.

5.6 Method of sepration of variables:For a generalised conservative system has the form S S + H q i , q = 0 t i If Hamiltonian does not explicitly contain time, one can linearly decouple time from rest of variables in S and we write S (q1, q2,.,qn, t) = S1(t) + V1 (q1, q2, qn) and its complete solution is of the form S = Et + V(q1, q2,., qn, 1,, n-1, E) [S = function of t + function of q S = Et + V(q1, q2,, qn, 1,., n1, E) H = 0 . Then Hamiltons Jacobi equation t

Example:- Write Hamiltonian for one-dimensional harmonic oscillator of mass m


& solve Hamilton-Jacobi equation for the same.

& Solution :- Let q be the position co-ordinates of harmonic oscillator then q is its
velocity and K.E., T = 1 & m q2 2

87

P.E., V = Lagrangian L = TV = The momentum is p=

1 kq2 2

[k is some constant]

1 1 & mq2 k q2 2 2

L & = mq & q j p m

& q=

Then, the Hamiltonian is H= & pi q i L

1 1 & & = p q mq 2 kq 2 2 2 =p H= p 1 p2 1 m 2 + k q2 m m 2

1 p2 1 2 + kq 2m 2 1 p2 1 2 1 p2 m 2 + kq = + kq 2 = Hamiltonian 2 m 2 2m (1)

Also for a conservative system, Total Energy = P.E. + K.E. =

H (q, p) =
Replacing p by

p 2 kq 2 + 2m 2

S in H, q
2

S 1 S kq 2 S S H q , q = 2m q + 2 . Then from t + H q i , q = 0, we get i S 1 S kq 2 + + =0 2 t 2m q


2

(2)

88

We separate variables S(q, t) = V(q) + S1(t) Putting in (2), we get 1 dV kq 2 dS1 (2) + + =0 2m dq 2 dt dS 1 dV kq 2 1= dt 2m dq 2 L.H.S. is function of t only and R.H.S. is function of q only, But it is possible only when each side is equal to a constant (E) (say) Let dS1 = E dt
2 2 2

S dS1 , S dV & = = dt t q dq

(i) (ii)

1 dV 1 2 & dq + 2 kq = E 2m (i) (ii) S1 = Et + constant dV 1 2 dq = 2m E 2 kq dV 1 = 2m E kq 2 dq 2 V(q) =


2

1 2 2m E kq 2 dq + cons tan t 2

Therefore, complete integral is S(q, t) = S1(t) + V(q) S(q, t) = Et +


1

1 2 2m E Kq 2 dq + cons tan t 2

Further by Jacobis theorem Here 1 = E

89

S = 1 1 1 = S E

S = 1 E 2m 2

1 = t +

dq
2 1 2 E Kq 2 1

1 = t +

m 2 , 2 K

dq 2E 2 q2 K dq
1

= t +

m K

E 2 K (q )

1 x dx = sin 1 2 2 a a x

m K sin 1 q 2E K K (t + 1 ) = sin 1 K q 2E m K k (t + 1 ) = sin q 2E m K 2E (t + 1 ) sin q(E, t) = K m The constants 1, E can be found from initial conditions The momentum is given by
= t+ p=

S (S1 ( t ) + V(q) ) = q q
=

V q
1

1 2 = 2m E Kq 2 2 = 2m 2E Kq 2 2

1 2

90

m 2E Kq 2 ,

which can be expressed as a function of t if we put q = q(t).

Example:- Central force problem in Polar co-ordinates (r, )


Here K.E, T= 1 . 2 &2 m r + r 2

r
P(r, )

( )

P.E., V(r) Then L = T V 1 & & L = m r 2 + r 2 2 V (r ) 2 pr =

L & r & r=

& & & & [q1 = r, q1 = r , q2 = , q 2 = ,pj = pr, p]


pr m

& pr = mr
p =

L & = mr 2 &

& p = 2 mr

Therefore, Hamiltonian is H=

& p q L
i i

1 & & & . & = p r r + p m r 2 + r 2 2 + V ( r ) 2 = p r& = p & pr mp 2 mr 2 p 2 r + p Q2 + V(r ) & 2 2 2 m mr 2mr 2m r

2 p2 p 2 p p2 r + 2 r 2 + V(r ) m mr 2m 2mr

p2 1 p2 = r + 2 + V (r ) 2 m mr =
2 1 2 p p r + 2 + V(r ) 2m mr

H J equation is

91

S +H=0 r
2 2 S 1 S 1 S + + 2 t 2m r r

+ V(r ) = 0
Using the Method of separation of variable, we have
S = S1(t) + R(r) + ()
2 2 dS1 1 dR 1 d = + 2 V (r ) dt 2m dr r d

L.H.S. is function of t and R.H.S. is function of r & but not of t, therefore it is not possible only where each is equal to constant = E (say) dS1 = E S1 (t) = Et + constant dt 1 d 1 dR and + 2 2m dr r d
2 2

V (r ) = E
2

r2 2m

1 d dR 2 2 r V (r ) + r E = 2m d dr

L.H.S is function of r only and R.H.S is function of only h2 (say ) So each = constant = 2m d =h d
2

( ) = h + constant

Then r equation gives

r 2 dR h2 2 2 r V(r ) + Er = 2m dr 2m
h2 dR = 2m(E V ) 2 r dr dR dr
2

= 2m(E V ) h 2 r 2

1 2

92

2m(E V ) h 2 r 2 dr + constant

Therefore, complete solution is S = S1 + R + S = Et + h + Now,

2m(E V ) h 2 r 2 dr + constant , is required solution

S = constant E t+

m dr h2 2m(E V ) 2 r

= 1 (say )

The other equation is S = constant h + 1 2

[2m(E V ) h r ]

( 2hr )dr
2

1 2 2 2

= 2 (say )

Example:-When a particle of mass m moves in a force field of potential V. Write


the Hamiltonian. Solution:- Here K.E. is T= 1 & & & m x 2 + y2 + z2 2

& x=

p px p & y , z = z and P.E. is V (x,y,z) & ,y = m m m

H=T+V H= 1 2 p x + p 2 + p 2 + V(x, y, z ) y z 2m

Example :- A particle of mass m moves in a force whose of potential in spherical


coordinates V is -cos/r2 . Write Hamiltonian in spherical coordinate (r, , ). Also find solution of H.J. equation. Solution:- L = 1 & & & m r 2 + r 2 2 + r 2 sin 2 2 V(r, , ) 2

93

pr = p =

L & = mr & r L & & = mr 2 , p = mr 2 sin 2 2 & &


2 2 p cos 1 2 p pr + + 2 2m r 2 r sin 2 r2

Hamiltonian is given by H=

Writing pr =

S S S , p = , p = r

Required Hamilton Jacobi equation is


2 2 2 S cos 1 S 1 S 1 S =0 + + 2 + 2 t 2m r r r sin 2 r2

(1)

Let S (t, r, , ) = S1(t) + S2(r) + S3() + S4() in (1)


2 2 S1 ds 4 1 dS 2 1 dS3 1 = + 2 + 2 2 2m dr t r d r sin d 2 cos + r2

L.H.S. is function of t only, R.H.S. is function of r, , and not of t, it is possible only when each is constant (= E) (say) dS1 = E dt
2

S1 = Et + cons tan t
2

dS 4 1 dS 2 1 dS 1 & + 2 3 + 2 2 2m dr r d r sin d Multiplying 2mr2 and rearranging terms, we get 1 dS 4 dS dS r 2 2mr 2 E = 3 sin 2 d dr d
2 2 2

cos =E r2

+ 2m cos

L.H.S. is function of r only and R.H.S. is function of and . It is possible only if each is equal to constant.

dS r 2 2 2mEr 2 = 1 dr

(2)

94

&

1 dS 4 dS 3 sin 2 d d

+ 2m cos = 1

(3)

S2 =

2mE +

1 dr + cons tan t r2
2

dS 4 d

dS = 2m cos sin 2 1 sin 2 sin 2 3 d

L.H.S. is function of whereas R.H.S. is function of and it is possible when each is equal to constant. dS 4 = 2 d and p = dS4 d
2

dS4 = 2 d

(4) (5)

S4 = p + constant
2 p = 2

[from (4) & (5)] [using (5)]

and S3 =

2 2m cos p cos ec 2 1 d + constant

The complete solution is S = Et +

2mE +

1 2 dr + 2m cos 1 p cos ec 2 d 2 r + p + constant

5.7 Lagranges Brackets


Lagranges bracket of (u, v) w.r.t. the basis (qj, pj) is defined as {u, v}q,p or (u, v)q,p = q j p j p j q j u v

u . v
j

Properties: (1) (u, v) = (v, u)


(2) (3) (4) (qi, qj) = 0 (pi, pj) = 0 (qi, pj) = ij

95

(u, v) =

q j p j p j q j p j q j q j p j = u v u v = -(v, u) u v u v j

(2) (qi, qj) =

qk . qk ' qk '
k

q pk j qi

[ Since qs and ps are independent

p k ' p k = o and 0] q j q i

= 0.

(3) Similarly we can prove that {pi, pj} = 0 (4) {qi, pj} =

qk . pk
k

qk pk = p j qi

q
k

q k p k = . i p j

ki

kj = ij

5.8 Invariance of Poissons Bracket under Canonical transformation:Poissons bracket is (u, v)q, p = u v u v p j p j q j j

q
j

The transformation of co- ordinates in a 2n dimensional phase space is called canonical if the transformation carries any Hamiltonian into a new hamiltonian system To show :- [F,G]q , p = [F, G]Q,P Poissons brackets is [F, G]q,p = F G F G i p i p i q i

q
i

If q, p are functions of Q & P then q = q (Q, P) & p = p (Q, P) and F & G will also function of (q, p), we have, G = G(Qk, Pk), we have

96

F G Q k G p k + q i Q k Pi Pk Pi [F, G]q, p = i F G Q k G Pk p Q q + P q i k i k i G F Q k F Q k Q k q i p i p i q i = i, k G F Pk F Pk + P q p p q k i i i i G G = F, Q q q ,p + [F, Pk ]q.p Pk i , k Q k

(1)

To find [F, Qk]q, p & [F, Pk]q, p Replacing F by Qi in (1) [Qi, G]q, p = G Q [Q , Q ]
i i ,k k k q ,p

G [Q i, Pk ]q,p PR

=0+ [Qi, G]q,p = G Pi

P
k k

ik

[G, Qi]q, p =
and [F, Qk]q, p =

G Pi F PR G q i (3) (2)

Replacing F by Pi in (1) [Pi, G] = and [F, Pk] = G i [G, Pi] =

F q k

Put these values from (2) and (3) in (1), we get: G F G F [F, G]q,p = P + P i ,k K K K K

97

= [F, G]Q,P
5.9 Poincare integral Invariant:-

Under Canonical transformation, the integral J = dq i dp i


S

(1)

Where S is any 2 D (surface) phase space remains Invariant


Proof :- The position of a point on any 2 D surface is specified completely by two

parameters, e.g. u, v Then q i = q i (u, v ) p i = p i (u, v ) (q i , p i ) du dv (u, v ) (2)

In order to transform integral (1) into new variables (u, v), we take the relation dqi dpi = (3)

q i (q i , p i ) = u where q i (u, v) v

p i u as the Jacobian. p i v
(4) (5)

Let Canonical transformation be Qk = Qk(q, p, t), Pk = Pk(q, p, t) then dQk dPk = ( K PK ) du dv (u, v )

if J is invariant under canonical transformation (4), then we can write

dq dp = dQ
i i S i S K

dPK (Q K , PK ) dudv

or

(u, v ) du dv = (u, v)
S i S K

(q i p i )

Because the surface S is arbitrary the expressions are equal only if the integrands are identicals, i.e.,

98

(u, v) = (u, v )
i K

(q i , p i )

(Q K , PK )

(6) In order to prove it, we would transform(q,p) basis to (Q, P) basesthrough the generating function F2(q, p, t), With this form of generating function, we have pi = F2 F & QK = 2 q i Pk = 2 F2 q k 2 F2 Pk + q .q u q .p u K i k i k = 2 F2 q k 2 F2 Pk + q .q v q .p v K i k i k p i u p i v 2 F2 q i q k 2F q i q k 2 F2 q i q k 2 F2 q i q k q k 2 F2 + u q i Pk q k 2 F2 + v q i Pk q k u + q k i ,k v q i u q i v Pk u Pk v 2 F2 q i Pk 2 F2 q i Pk Pk u Pk v

p i F2 = u u q i and

p i F2 = v v q i

q i (q i , p i ) Now, = u q i (u, v ) i i v q i u q i v q i u q i v

=
i ,k

=
i ,k

q i 2 F u L. H. S of (6) = q i i ,k q i q i v

q k u q k v q i Pk 2 F u u (7) + q i Pk q i Pk i,k v v We see that first term on R.H.S. is antisymmetric expression under interchange of i and k, its value will be zero,

99

i.e., q k 2F u = q k q k q i k ,i v q i q k 2 F u u = q k q i q i q k k ,i v v q i q k 2 F u or q q q i quk = 0 i ,k i k v v Similarly replacing q by P we have from (8) Pi Pk 2 F u u P P Pi Pk = 0 i ,k i k v v Now equation (7) can be written as Pi Pk (q p ) F u u (ui, vi ) = P P Pi Pk i i ,k i k v v q i Pk 2 F u u + q i Pk i ,k q i Pk v v
2

q i 2 F u q q q i i ,k i k v

q k u q k v

q i u q i v

(8)

i ,k

2 F2 Pk Pi 2F Pk Pi
F2 u Pk F2 v Pk

Pi 2 F2 + u Pk q i Pi 2 F2 + v Pk q i
Pk u Pk v

q i u q i v

Pk u Pk v

=
k

100

Put

F2 = Qk Pk =
k

Q k u Q k v

Pk u = (Q k , Pk ) = R.H.S of (6). (u, v ) Pk k v

Which proves that integral is invariant under canonical transformation.

5.10 Lagranges bracket is invariant under Canonical transformation:The Lagranges bracket of u & v is defined as

{u, v}q ,p = q i
i

p i q i p i u v v u q i u p i u q i v p i v is invariant under Canonical transformation.

Since

(u, v )
i

(q i p i )

So Lagranges bracket is also invariant under canonical transformation

101

Lesson-6
AB be a rod APB = X

Attraction and Potential

6.1. Attraction of a uniform straight rod at an external point:P p Y

Q xQ

Let m be the mass per unit length of a uniform rod AB. It is required to find the components of attraction of the rod AB at an external point P. MP = p Consider an element QQ of the rod where MQ = x QQ = dx MPQ = , In MPQ , tan = cos = PQ = MQ x = MP p MP p = PQ pQ PQ = p sec (**) x = p tan (*)

p cos

Mass of element QQ of rod = m dx = mp sec2 d The attraction at P of the element QQ is = (using*) mass = mp sec 2 d

(dis tan ce)2

(PQ )2

along PQ

Therefore, Force of attraction at P of the element QQ is

102

mp sec 2 d = p 2 sec 2 = m d p along PQ

[using(**)]

(1) Let MPA = and MPB = f=

p d

let X and Y be the components of attraction of the rod parallel & r to rod, then X=

p sin d

&

Y= X=

p cos d

m [ cos ] = m [cos cos ] p p = m 2 sin 2 sin 2 p (2)

and

Y=

m [sin ] = m [sin sin ] p p = m + 2 cos 2 sin 2 p (3)

Resultant force of Attraction R is given by R= R= = X2 + Y2 2m sin p 2 [using 2&3]

APB 2m sin 2 p X Resultant R makes angle tan1 Y

103

or

1 ( + ) with PM 2

1 + 1 X tan = tan tan 2 Y

i.e. it acts along bisector of angle APB. Also X= m m PB PA p p cos = PB , cos = PA & u sin g(2)

Cor :- If the rod is infinitely long, the angle APB is two right angles & Resultant
attraction = 2m r to the rod. p

6.2 Potential of uniform rod :By definition, the potential at P is given by V= V= m

PQ dx
mp sec 2 p sec d

= m sec d

= m log tan + 4 2

= m log tan + log tan + 4 2 4 2 tan 2 + 4 = m log tan 2 + 4

6.3 Potential at a point P on the axis of a Uniform circular disc or plate:We consider a uniform circular disc of radius a & P is a pt. on the axis of disc & the pt. P is at a distance r from the centre 0, i.e., OP = r, OQ = x, PQ = r 2 + x 2 let us divide the disc into a number of concentric rings & let one such ring has radius x & width dx Then, Mass of ring is = 2x dx

104

where density of material of disc = mass/Area Therefore, Potential at P due to this ring is given by ,dv = P

2x dx r2 + x2

r2 + x2

r OP = r Q O Q

Hence, the potential at P due to the whole disc is given by V = 2


a

x dx x2 + r2

V=

1 2 2x (x 2 + r 2 ) 2 dx 2 0

V = 2 Let Mass of disc = M = a2 = Then V = disc. 2m a2 M a2


2

[a

+ r2 r

[a

+ r 2 r is requiredpotential at any pt P which lies on the axis of

6.4 Attraction at any point on the axis of Uniform circular disc :Here radius of disc = a OP = r, OQ = x PQ =

x2 + r2

105

We consider two element of masses dm at the two opposite position Q and Qas shown. Now element dm at Q causes attraction on unit mass at P in the direction PQ. Similarly other mass dm at Q causes attraction on same unit mass at P in the direction P Q and the force of attraction is same in magnitude. These two attraction forces when resolved into two direction one along the axes PO and other at right angle PO. Components along PO are additive and component along perpendicular to PO canceling each other Mass of ring = 2x dx Attraction at P due to ring along PO is given by ( dm )cos df = (PQ )2 cos Q.2x dx r.2xdx df = = along PO (PQ )2 (PQ )3 2.rx dx [ cos = r inOPQ ] PQ

(r

+ x2

3 2

Therefore, the resultant attraction at P due to the whole disc along PO is given by
a f = r (2x ) r 2 + x 2

3 2

dx
a

= r 2 x 2 + r 2

1 2

1 1 = 2r along PO a2 + r2 r Let M = mass of disc of radius a =a2 = M a2

2M r f = 2 1 a a2 + r2

106

2M [1 cos ] a2

Where is the angle which any radius of disc subtends at P

Particular cases :1. If radius of disc becomes infinite, then =


and 2M f = 2 1 cos 2 a = 2M = constant [here, it is independent of position of P] a2 2

2. When P is at a very large distance from the disc, then 0


2M Therefore, f = 2 (1 cos 0) a =0

6.5 Potential of a thin spherical shell :We consider a thin spherical shell of radius a & surface density let P be a point at a distance r from the center O of the shell. We consider a slice BBCC in the form of ring with two planes close to each other and perpendicular to OP. Area of ring (slice) BBCC =2BDBB where Radius of ring, width of ring, = 2 a sin ad = 2a sin d Hence, Potential at P due to slice (ring) is 2a 2 sin d dV = x C C (1)
2

BD = a sin BB= a d

x r P

Therefor,Mass of slice (ring) is O

107

Now, from BOP, BP2 = OP2 + OB2 2OP.OB cos x2 = r2 + a2 2ar cos differentiating 2x dx = 2ar sin d x dx = sin d ar Putting in (1), we get dV = = 2a 2 x dx x.ar (2)

2adx r

Therefore, Potential for the whole spherical shell is obtained by integrating equation (2), we have V= = 2a dx r

2a dx r

Now, we consider the following cases :Case(i) The point P is outside the shell. In this case, the limit of integration extends
from x = (r a) to x = (r + a) Hence V= 2a dx r r a = 4a 2 r
r+a

V=

Here, Mass of spherical shall = 4a2 Then V = M r

Case(ii) When P is on the spherical shell, then limits are from x = 0 to x = 2a


(here r = a)

108

2a Then V = dx a 0 = 4a 2 M = a a M a

2a

Case(iii) When P is inside the spherical shell, limit are from x = (a r) to (a + r)


V = 4a =

6.6 Attraction of a spherical shell


Let us consider a slice BBCC at point P , the attraction due to this slice is 2a 2 sin d df = along PB x2 The resultant attraction directed along PO is given by 2a 2 sin d df = cos x2 We know that sin d = In BDP, cos = xdx ar

PD r a cos = . PB x

2a 2 xdx r a cos df = x ar.x 2 We know that x2 = a2 + r2 2arcos x2 a2 + r2 = 2r2 2arcos x2 a2 + r2 = r a cos 2r 2a 2 xdx x 2 a 2 + r 2 Then, d f = 2r , x ar.x 2 = a x 2 a 2 + r 2 r2 x2 dx

109

a r 2 + a 2 1 + r2 x2

dx

Hence the attraction for the whole spherical shell is obtained by integration Therefore, a r 2 a 2 f = 2 1 + dx r x2

Now we consider the following cases depending upon the position of P

Case(i) When point P is inside the shell, the limits of integration are x = (r a) to
(r + a)

f =

r +a a r 2 a 2 1 + r 2 r a x2

dx
r +a

f =

a 1 x + r 2 a 2 2 r x r a

4a 2 M = 2 r2 r dx

Case(ii) When pt. P is on the shell, the limit of integration are x = 0 to 2a


a 2a r 2 a 2 f = 2 1 + r 0 x2

Here integration is not possible (due to second term is becoming indeterminant), because when P is on the shell, then r = a; x = 0 Hence to evaluate the integral, we consider that pt. P is situated not on the surface but very near to the surface Let r = a +, where is very small
2a + a 2 a + (a + )2 a 2 f = 2 dx + dx r x2 2a +

Then attraction is a = 2 2a + r

2a dx x2

110

a 1 2a + 2a 2 r x

2a +

= = =

a 2a 2a + 2a 2 2a + r 2a 2 2 2 2a + as o, then r = a r 4a 2 M = 2 2 a a

Case (iii) Poin. P is inside the shell, limits are x = a r to a + r


a a + r r 2 + a 2 f = 2 1 + dx r a r x2 = a 1 = 0 x r 2 a 2 2 r x a r

a +r

So, there is no resultant attraction inside the shell.

6.7 Potential of a Uniform solid sphere:- A uniform solid sphere may be


supposed to be made up of a number of thin uniform concentric spherical shells. The masses of spherical shells may be supposed to be concentric at centre O.

Case I :- At an external point

Therefore the potential due to all such shells at an external point P is given by V= m1 m 2 + + r r 1 M (m1 + m 2 + ...) = r r

where m1, m2 etc are the masses of shells. V=

111

where M is the mass of solid sphere.

Case II:- The point P is on the sphere.


In case I, put r = a V= M , where a = radius of sphere a x 0

Case III:- At an internal point. Here point P is


considered to be external to solid sphere of radius r & internal to the shell of internal radius r, external radius = a. Let V1 = potential due to solid sphere of radius r

a P

V2 = potential due to thick shell ofinternal radius r and external radius a Then V1 = = mass of sphere of radius r r 4 r 3 4 2 = r 3 r 3

To calculate V2
We consider a thin concentration shell of radius x & thicknes dx. The potential at P due to thin spherical shell under consideration is given by 4x 2 dx = 4x dx x Hence for the thick shell of radius r & a, the potential is given by V2 = 4 x dx
r a

a2 r2 2 2 = 4 2 = 2(a r ) Therefore, the potential at P due to given solid sphere. V = V1 + V2 = 2 (3a2 r2) 3 4 3 a 3

where M = Mass of given solid sphere =

112

3M 4a 3 2 3M M . (3a 2 r 2 ) = 3 (3a2 r2) 3 3 4 a 2a

Hence V =

6.8 Attraction for a uniform solid sphere Case I : At an external point


m m F = 21 + 22 + . r r M F= 2 , r M = m1 + m2 +. x 0 r P

a M = Mass of sphere and m1 , m2. Are masses of concentric spherical shells

Case II: At a point on the sphere,


Here we put r = a in above result M We get F = 2 a

Case III: At a point inside the sphere.


The point P is external to the solid sphere of radius r and it is internal to thick spherical shell of radii r and a. And we know that attraction (forces of attraction) at an internal point in case of spherical shell is zero. Hence the resultant attraction at P is only due to solid sphere of radius r and is given by mass of sphere of radius r F= r2 = If M = 4 r 3 4 = r 3 r2 3

4 3 3M a = 3 4a 3 Mr Then F = 3 a

113

6.9 Self attracting systems:- To find the work done by the mutual attractive forces
of the particles of a self-attracting system while the particles are brought from an infinite distance to the positions, they occupy in the given system. System consists of particles of masses m1, m. at A1, A2.. etc. in the given system A. We first being m1 from infinity to the position A1. Then the work done in this process is zero. Since there is no particle in the system to exact attraction on it next m2 is brought from infinity to its position A2. Then the work done on it by m1 is = potential of m1 at A2 m2 = mm m1 m2 = 1 2 r12 r12 m1 & A1 m2 & A2 m3 & A3

where r12 = distances between m1 & m2 (r12 = r21) Then these two particles m1 and m2 attracts the third particle. Work done on m3 by m1 & m2 is = m1m 3 m 2 m 3 + r13 r23 m1m 4 m 2 m 4 m 3 m 4 + + r14 r24 r34

when m4 is brought from infinity to its position A4 =

Hence the total work done in collecting all the particles from rest at infinity to their positions in the sysem A. = = Let V1 = m1m 2 m1m 3 m 2 m 3 + + + .. r r12 r23 13

ms m t , where summation extends to every pair of particles. rst

m 2 m3 + + r12 r13

= potential at A1 of m2, m3 .. s one V2 = potential at A2 of m1, m3, m4.

114

= V3 =

m1 m 3 + + r21 r23 m1 m 2 m 4 + + m13 r23 r43

ms m t 1 = [V1 m1 + V2 m2 ++ V3m3] rst 2 1 mv 2

Total work done =

This represents the work done by mutual attraction of the system of particles. If the system from a cont. body, then work done will be = 1 v dm 2

Conversely (if particle is scattered) the work done by the mutual attraction forces of the system as its particles are scattered at infinite distance from confinguration A, then work done = 1 1 mv = 2 v dm 2

We can find the work done as the body changes from one configuration A to another configuration B. The work done in changing of its from A to state at infinity + work done in collecting particles in a state at infinity to configuration B = 1 1 V dm + 2 V' dm' 2 A B

AB

Example. A self attracting sphere of uniform density & radius a changes to one
of uniform density & radius b. Show that the work done by its mutual attractive forces is given by 3 2 1 1 M 5 b a where M is mass of sphere.

Solution: Here the work done by mutual attractive forces of the system. As the
particle which constitute the sphere of radius a are scattered to infinite distance

115

w1 =

1 v dm 2

We consider a point within the system at a distance x. The potential at this point within the spheres V= 2 (3a 2 x 2 ) 3

Let us now consider at this point, a spherical shell of radius x & thickness dx, then dm = 4 x2 dx V dm = = 2 (3a2 x2) 4x2 dx 3 8 2 2 2 x (3a2 x2) dx 3
a

8 2 2 2 2 2 v dm = 3 x (3a x ) dx 0 8 2 2 2 x3 x5 = 3a 3 3 5 0 = = 8 2 2 5 a 5 8 2 2 4a 5 a = 3 5 3 5 32 2 2 5 a 15 4 3 3m a = 3 4 a 3 6 m2 5 a
a

M = Mass of sphere of radius a =

v dm =
W1 =

1 3 m2 v dm = 2 5 a

Similarly if W2 is work done in bringing the particle to the second configuration (a sphere of radius b)

116

1 3 m2 Then W2 = v' dm' = 5 b 2 Total work done is given by W = W1 + W2 = 3 2 1 1 m 5 b a

6.10 Laplaces equation for potential


Let V be the potential of the system of attracting particles at a point P(x, y, z) not in contact with the particles so that V= m r

(1)

where m is the mass of particle at P0(a,b,c) r = distance of P from the P0, and r = (x a) + (y b) + (zc) (1)
2 2 2 2

P (2)

P0

m r m (r a ) V = 2 = 2 r x r r2 x

r r x a (2) 2r x = 2( x a ) x = r and V m ( y b) = y r3 m ( z c) V = r3 z 2V [ m (x a) r3] = 2 x x = m(x a) (3 r4) m r3 (1) = m 3 and 2V = y 2 (x a) 2 m 3 5 r r r x

m ( y b) 2 m r5 r3

117

m ( z c) 2 m 2V = 3 3 ss 2 5 r r z 2V 2V 2V + + =0 x 2 y 2 z 2

which is Laplace equation. V potential dv = small volume elemnt dm = dv So V= dv r

V 1 r = 2 dv x r x

6.11 Poissons equation for potential


Let the point P(x, y, z) be in contact (inside) the attracting matter. We describe a sphere of small radius R & centre (a, b, c) contains the point P. = density of material (sphere) Since the sphere we describe is very small, therefore we cosndier the matter inside this sphere is of uniform density . So potential at P may be due to (i) (ii) the matter inside the sphere the matter outside the sphere. R (a,b,c)

V1 = contribution towards potential at P by the matter outside the sphere V2 = contribution towards potential at P by the matter inside the sphere. Since the point P is not in contact with the matter outside the sphere. Therefore by Laplace equation 2V1 = 0. Here V2 = potential at P(x, y, z) inside the sphere of radius R. V2 = 2 (3R2 r2) 3

118

where r2 = (x a)2 + (y b)2 + (z c)2 V2 2 r 2 r(x a) = 2r = (2) x 3 x 3 r = 4 (x a) 3

2 V2 4 = 3 x 2 2 V2 4 , = 3 y 2 2V 4 = 3 z 2

Similarly

2 V2 2 V2 2 V2 + + = 4 x 2 y 2 z 2 2 V2 = 4 2V = 2 V 1 + 2V 2 2V 2V 2V + 2 + 2 = 2V = 4 2 x y z

Since total potential V = V1 + V2

This equationis known as Poissons equation

6.12 Equipotential Surfaces


The potential V of a given attracting system is a function of coordinates x,y,z. The equation V(x, y, z) = constant represents a surface over which the potential is constant. Such surfaces are known equipotential surfaces. Condition that a family of given surfaces is a possible family of equipotential surfaces in a free space. To find the condition that the equation f(x, y, z) = constant may represent the family of equipotential surface. If the potential V is constant whenever f(x, y, z) is constant, then there must be a functional relation between V and f(x, y, z) say

119

V = {f(x, y, z)} V = (f) V f = (f) x x 2V = (f) x 2 2V = (f) y 2 2f f + (f) 2 x x f 2f + (f) y y 2


2 2 2

2V 2f f = (f) + (f) 2 z 2 z z Adding 2f 2f 2f 2V = (f) 2 + 2 + 2 + (f) x y z But in free space, 2V = 0 2f 2f 2f + + x 2 y 2 z 2 f f f + + x y z


2 2 2

f 2 f 2 f 2 + + y y x

' ' (f ) = a function of f ' (f )

= (f) (say) expressed in terms of f(x, y, z). Then V = (f), where ' ' (f ) + (f) = 0 ' (f )

(1)

This is the necessary condition and when it is satisfied, the potential V can be

Integrating, log (f) = log A (f)df ' (f ) log = (f) df A (f) =Ae(f)df

Again integrating,

120

V = (f) = A e ( f ) df df + B which is required expression in terms of f(x, y, z) for V.

(2)

Example :- Show that a family of right circular cones with a common axis & vertex
is a possible family of equipotential surfaces & find the potential function.

Solution : Taking axis of z for common axis. The equation of family of cones is
f(x, y, z) = f 2x = , x z 2 f 2 y = , y z 2 x 2 + y2 = constant z2 f 2 = 2 2 x z 2f 2 = 2 2 y z (3)

f = (x2 + y2) (2) (z3) z 2f = 6(x2+ y2) z4 2 z Therefore condition (1) becomes

' ' (f ) = 2 2 2 ' (f ) f f f + + x y z 2 2 6( x 2 + y 2 ) + 2+ 2 z4 = z2 z 2 4x 4y 4( x 2 + y 2 ) 2 + 4 + z4 z z6 = = 2z 2 + 2z 2 + 6( x 2 + y 2 ) z6 z4 4 z 2 ( x 2 + y 2 ) + 4( x 2 y 2 ) 2 4z 2 + 6( x 2 + y 2 ) [ 4z 2 ( x 2 + y 2 ) + ( x 2 + y 2 ) 2 ]

2f 2f 2f + + x 2 y 2 z 2

121

3( x 2 + y 2 ) 2z 2 + z2 = 2 2 2 x 2 + y2 4 x +y 4z + z2 z2 = 2 + 3f 2 + 3f = = function of f 2 2(f + f ) 2f (f + 1)

' ' (f ) 2 + 3f = [function of f 0, condition (1) is satisfied] ' (f ) 2f (1 + f ) ' ' (f ) 2 + 3f + =0 ' (f ) 2f (1 + f ) ' ' (f ) 1 1 + + =0 ' (f ) f 2(1 + f )

Integrating, log (f) + log f + Put log (f) = log (f) = C f 1+ f 1 log (1 + f) = log C 2

C f 1+ f

d C = df f 1 + f

d = f
f = tan2

C df + C 1+ f

df = 2 tan sec2 d = C = C 2 tan sec 2 d tan 2 . 1 + tan 2 + C

2 sec 2 . d + C' tan sec

122

= 2C

tan d

sec

+ C

= 2C cos ec d + C or V = (f) = 2C log (cosec cot ) + C V = (f) = 2C log tan + C is the required potential function. So V is 2

constant when is constant.


6.13 Variation in attraction in crossing a surface on which there exist a thin layer of attracting matter.

Let P1 and P2 be two points on the opposite side of surface. surface density of small circular disc of the surface between P1 and P2. For potential at surface, V1 = V2 V2 V1 = 4 n n P1 P2

To find the attraction of matter, when the potential is given at all points of space, then Poissons equation 2V = 4 gives the volume density of matter = 1 2 V 4

If potential is given by different functions V1, V2 on opposite side of a surface S, then surface density is given by = 1 V2 V1 4 n n

Example. The potential outside a certain cylindrical boundary is zero, inside it is

V = x3 3xy2 9x2 + 3ay2. Find the distribution of matter.

123

Solution : Since V2 = outside potential and V1 = Inside potential

Here

V2 = 0

We find the boundary. Since the potential is continuous across the boundary & zero outside the boundary. The boundary may be given by x3 3xy2 ax2 + 3ay2 = 0 or (x a) (x2 3y2) = 0 (x a) (x + 3 y) (x 3 y) = 0 y

AB is equation of line x = a OB is equation of line x + OA is equation of line x 3y =0 3y=0 0 n P


60 30

The section is an equilateral OAB of height a. V1 = 3x2 3y2 2ax x V1 = 6xy + 6ay y 2 V1 = 6x 2a x 2 2 V1 = 6x + 6a y 2 2 V1 =0 z 2 so that inside the region, = = 1 2 V1 4 1 [4a] 4 = a

30 a 30

P M
B

and outside, = 0 since V2 = 0 At P on AB (x = a),

124

= = =

1 V2 V1 x x =a 4 x 1 [ 0 3x2 + 3y2 + 2ax] 4 3 a2 1 2 2 [3y a 2 ] = y 4 4 3 (1)

In OAM, OA2 = OM2 + AM2 OA2 = a2 + 1 (OA)2 4 (OA)2 = 4 2 a 3 1 2 a 3 (2)

3 (OA) 2 = a2 4 (2 MA)2 = 4 2 a 3

(MA)2 =

from (1) & (2), we have = = = 3 [MA2 MP2] 4 3 (MA + MP) (MA MP) 4 3 (PB) (AP) 4 3 y)

At P on OA (x = = =

1 V1 4 n V1 V1 1 sin 30 x + cos 30 y 4 x= 1 1 V1 3 V1 + 4 2 x 2 y x =

3y

3y

1 3 2 3 2 2 x 2 y + ax 3 3xy + 3 3ay 4 x=

3y

125

= =

x 1 3 2 3 x2 + ax 3x 2 + 3 3 a. x + 2 3 4 2 3 1 x(a x).

6.14 Harmonic functions

Any solution of Laplace equation 2 V = 0 in x, y, z is called Harmonic function or spherical harmonic. i.e. 2V 2V 2V + + =0 x 2 y 2 z 2

If V is a Harmonic function fo degree n, then p q t is a harmonic function of degree n p q t. x p y q z t Now 2V = 0 [Laplace equation] p times w.r.t. x q times w.r.t. y t times w.r.t. z So q q t 2 p q t = 0 x y z

6.15 Surface and solid Harmonics :- In spherical polar coordinates (r, , )

Laplace equation is 1 1 2V V 2 V =0 + sin + r sin 2 2 r r sin Let V = rn Sn where Sn is independent of r or Sn(, ). 2 V 2 n r r (r Sn ) = r r r r = = 2 [r Sn n rn1] r n+1 [Sn n rn+1] = nSn (r ) r r (1)

126

= n (n + 1) rn Sn (1) n (n + 1) rn Sn + n(n + 1) rn Sn + 2S n 1 1 n S n rn =0 sin .r + sin sin 2 2 S rn sin n sin r n 2S n + =0 2 2 sin

n(n +1) Sn +

S 1 1 2S n =0 sin n + sin sin 2 2 Sn 2Sn 1 2S n + + =0 2 sin 2 2 (2)

n(n +1) Sn + cot

If cos = N(n +1) Sn + S 1 2S n (1 2 ) n + =0 1 2 2 (3)

A solution Sn of equation (2) is called a Laplace function or a surface harmonic of order n. Since n(n+1) remains unchanged when we write (n +1) for n. So there are two solutios of (1) of which Sn is a factor namely rn Sn and rn1 Sn. These are known as solid Harmonic of degree n & (n +1) respectively.
Remark:

1.

If U is a Harmonic function of degree n, then

U r
2 n +1

is also Harmonic

function. U = rn Sn so that U r 2 n +1 = r n Sn S = n n1 = Sn r(n+1) 2 n +1 r r + xyz is also Harmonic. r7

which is Harmonic. Let xyz 3rd degree is a solutionof Laplace equation, then 2.

If U is a Harmonic function of degree (n +1), then Ur2n+1 is also a U = rn1 Sn

Harmonic function, are may write

127

so that r2n+1 U = r2n+1 rn1 Sn = rn Sn which is Harmonic.


6.16 Surface density in terms of surface Harmonics

The potential at any point P due to a number of particles situated on the surface of sphere of radius a can be ut in the form V1 = V2 =

n =0

rn Un , when r < a a n +1 when r > a

(1) (2)

an r n+1 Un

where Un denotes the sum of a number of surface Harmonics (for each particle) & therefore itself a surface harmonic. We assume (1) & (2) to represent potential of a certain distribution of mass & want to find it (density) on the surface. Here U1 is Harmonic 2V1 = 0, 2V2 = 0 V V 4 = 2 1 r r =a r = 1 V1 V2 4 r r r =a U a n (n + 1) 1 U n n r n 1 + n n +2 4 a n +1 r r =a Here on the surface of sphere, density is given by

U n a n (n + 1) 1 U n n a n 1 + = 4 a n +1 a n+2 = = (n + 1) 1 n Un a 2 + Un a 2 4

(2n + 1) U n 4a 2

(3)

If potential is given by (1) & (2), then surface density is given by (3).

S-ar putea să vă placă și